69
LỜI CẢM ƠN Lời đầu tiên em xin bày tỏ lòng kính trọng và biết ơn sâu sắc đến Thầy giáo Thạc sĩ Nguyễn Văn Hà. Thầy đã tận tình hƣớng dẫn và giúp đỡ em thực hiện đề tài, hoàn thành khóa luận. Em cũng xin chân thành gửi lời cảm ơn tới các Thầy cô giáo khoa Toán Trƣờng ĐHSP Hà Nội 2 đã giúp đỡ em hoàn thành khóa luận này. Do đây là lần đầu làm quen với công tác nghiên cứu khoa học, hơn nữa thời gian và năng lực còn có những hạn chế, mặc dù đã có nhiều cố gắng nhƣng không tránh khỏi những thiếu sót. Em rất mong nhận đƣợc sự góp ý của các Thầy cô và các bạn để khóa luận đƣợc hoàn thiện hơn. Em xin chân thành cảm ơn! Hà Nội, ngày 10 tháng 5 năm 2014 Sinh viên Nguyễn Thị Lan Hƣơng

Sinh viên Nguyễn Thị Lan Hƣơng · PDF fileLỜI CẢM ƠN Lời đầu tiên em xin bày tỏ lòng kính trọng và biết ơn sâu sắc đến Thầy giáo Thạc sĩ Nguyễn

  • Upload
    vanhanh

  • View
    218

  • Download
    2

Embed Size (px)

Citation preview

Page 1: Sinh viên Nguyễn Thị Lan Hƣơng · PDF fileLỜI CẢM ƠN Lời đầu tiên em xin bày tỏ lòng kính trọng và biết ơn sâu sắc đến Thầy giáo Thạc sĩ Nguyễn

LỜI CẢM ƠN

Lời đầu tiên em xin bày tỏ lòng kính trọng và biết ơn sâu sắc đến Thầy

giáo Thạc sĩ Nguyễn Văn Hà. Thầy đã tận tình hƣớng dẫn và giúp đỡ em thực

hiện đề tài, hoàn thành khóa luận.

Em cũng xin chân thành gửi lời cảm ơn tới các Thầy cô giáo khoa Toán

Trƣờng ĐHSP Hà Nội 2 đã giúp đỡ em hoàn thành khóa luận này.

Do đây là lần đầu làm quen với công tác nghiên cứu khoa học, hơn nữa

thời gian và năng lực còn có những hạn chế, mặc dù đã có nhiều cố gắng nhƣng

không tránh khỏi những thiếu sót. Em rất mong nhận đƣợc sự góp ý của các Thầy

cô và các bạn để khóa luận đƣợc hoàn thiện hơn.

Em xin chân thành cảm ơn!

Hà Nội, ngày 10 tháng 5 năm 2014

Sinh viên

Nguyễn Thị Lan Hƣơng

Page 2: Sinh viên Nguyễn Thị Lan Hƣơng · PDF fileLỜI CẢM ƠN Lời đầu tiên em xin bày tỏ lòng kính trọng và biết ơn sâu sắc đến Thầy giáo Thạc sĩ Nguyễn

LỜI CAM ĐOAN

Với sự hƣớng dẫn tận tình của Thạc sĩ Nguyễn Văn Hà, sau một thời gian

nghiên cứu và thực hiện đề tài, tôi đã hoàn thành khóa luận của mình. Để thực

hiện đề tài, tôi đã sử dụng và tham khảo các kết quả của các nhà khoa học, một

số sách tham khảo về bất đẳng thức. Tôi xin cam đoan khóa luận với đề tài “ Dạy

học bài toán chứng minh bất đẳng thức ở trƣờng trung học phổ thông theo

phƣơng pháp dạy học tích cực” là nghiên cứu của tôi, không trùng lặp với bất kỳ

khóa luận nào khác.

Hà Nội, ngày 10 tháng 5 năm 2014

Sinh viên

Nguyễn Thị Lan Hƣơng

Page 3: Sinh viên Nguyễn Thị Lan Hƣơng · PDF fileLỜI CẢM ƠN Lời đầu tiên em xin bày tỏ lòng kính trọng và biết ơn sâu sắc đến Thầy giáo Thạc sĩ Nguyễn

DANH MỤC CHỮ VIẾT TẮT VÀ KÍ HIỆU TOÁN HỌC

BBT: Bảng biến thiên

GV: Giáo viên

GTNN: Gía trị nhỏ nhất

HS: Học sinh

PPDH: Phƣơng pháp dạy học

THPT: Trung học phổ thông

VT: Vế trái

abcfacbfcbafcbafcyc

,,,,,,,,, .

Page 4: Sinh viên Nguyễn Thị Lan Hƣơng · PDF fileLỜI CẢM ƠN Lời đầu tiên em xin bày tỏ lòng kính trọng và biết ơn sâu sắc đến Thầy giáo Thạc sĩ Nguyễn

MỤC LỤC

MỞ ĐẦU ....................................................................................................... 1

CHƢƠNG 1. CƠ SỞ LÝ LUẬN VÀ THỰC TIỄN ..................................... 3

1.1. Phƣơng pháp dạy học tích cực ............................................................... 3

1.2. Lý luận chung về bài toán ...................................................................... 7

1.3. Những kiến thức cơ bản liên quan đến bất đẳng thức ............................ 9

CHƢƠNG 2. VẬN DỤNG PHƢƠNG PHÁP DẠY HỌC TÍCH CỰC

VÀO DẠY HỌC BÀI TOÁN CHỨNG MINH BẤT ĐẲNG THỨC Ở

TRƢỜNG THPT ......................................................................................... 13

2.1. Phƣơng pháp tích cực vận dụng trong chứng minh bất đẳng thức ...... 13

2.2. Vận dụng phƣơng pháp tích cực trong chứng minh bất đẳng thức ...... 18

2.2.1. Phƣơng pháp chứng minh bất đẳng thức dựa vào các bất đẳng

thức cơ bản .................................................................................................. 18

2.2.2. Phƣơng pháp chứng minh bất đẳng thức dựa vào hàm số ................ 28

2.2.3. Phƣơng pháp chứng minh bất đẳng thức dựa vào cách đặt ẩn

phụ ............................................................................................................... 47

2.2.4. Phƣơng pháp chứng minh bất đẳng thức bằng cách kết hợp

nhiều phƣơng pháp khác nhau .................................................................... 51

2.2.5. Khai thác các bài toán chứng minh bất đẳng thức ............................ 54

KẾT LUẬN ................................................................................................. 64

TÀI LIỆU THAM KHẢO ........................................................................... 65

Page 5: Sinh viên Nguyễn Thị Lan Hƣơng · PDF fileLỜI CẢM ƠN Lời đầu tiên em xin bày tỏ lòng kính trọng và biết ơn sâu sắc đến Thầy giáo Thạc sĩ Nguyễn

KHÓA LUẬN TỐT NGHIỆP ĐẠI HỌC Nguyễn Thị Lan Hương – K36B SP Toán

1

MỞ ĐẦU

1. Lý do chọn đề tài.

Toán học có vai trò rất quan trọng trong quá trình hình thành và phát triển

tƣ duy của học sinh. Trong toán học phổ thông, các bài toán bất đẳng thức chiếm

vị trí đặc biệt quan trọng, nó xuất hiện hầu hết trong các kỳ thi tuyển sinh các

cấp, kỳ thi chọn học sinh giỏi toán cấp tỉnh, cấp Quốc Gia, Quốc Tế… và thƣờng

xuất hiện dƣới dạng là bài toán khó nhất trong đề. Đề bài của bài toán bất đẳng

thức tuy đƣợc phát biểu hết sức ngắn gọn, sáng sủa, đẹp đẽ nhƣng học sinh lại

gặp rất nhiều khó khăn khi đi tìm lời giải và càng khó khăn hơn trong kỹ năng

khai thác chúng. Trƣớc những vấn đề trên tôi nhận thấy cần đi tìm những thuật

giải, những hƣớng đi cụ thể để giải quyết các vấn đề đó. Nhƣng chúng ta đã biết

không có một chìa khoá vạn năng nào có thể “mở khoá” đƣợc mọi bài toán.

Trong khi đó việc giảng dạy toán học, làm cho học sinh giải quyết đƣợc vấn đề

đặt ra của bài toán một cách sáng tạo, hoàn chỉnh cũng nhƣ khai thác đƣợc các

bài toán đó là rất cần thiết.

Bất đẳng thức là một trong những nội dung hay của Toán phổ thông, cũng

là một nội dung quan trọng nhằm rèn luyện trí tuệ cho học sinh. Nhìn bất đẳng

thức dƣới nhiều phƣơng diện khác nhau giúp học sinh linh hoạt trong lựa chọn

hình thức thể hiện nội dung này. Điều đó kích thích tƣ duy biện chứng, tƣ duy

sáng tạo cho các em.

Tuy nhiên, bất đẳng thức cũng là một nội dung khó, nếu không đổi mới

phƣơng pháp dạy học thì có thể dẫn đến tình trạng truyền thụ một chiều. Định

hƣớng đổi mới phƣơng pháp dạy học hiện nay là tích cực hóa việc học của ngƣời

học. Để giải quyết mâu thuẫn trên đây ngƣời thầy cần tăng cƣờng giao lƣu giữa

thầy và trò trong quá trình dạy học, phát huy tính tích cực, tự giác, chủ động, tƣ

duy sáng tạo của học sinh; phù hợp với đặc điểm của từng lớp học; bồi dƣỡng

phƣơng pháp tự học, rèn luyện kỹ năng vận dụng kiến thức vào thực tiễn; tác

động đến tình cảm, đem lại niềm vui, hứng thú học tập cho các em. Có nhƣ vậy

mới có thể vừa tích cực hóa đƣợc việc học của ngƣời học vừa rèn luyện đƣợc

tính linh hoạt nhìn nhận một vấn đề theo nhiều phƣơng diện khác nhau, nhằm

nâng cao khả năng tƣ duy, phát triển trí tuệ đồng thời bồi dƣỡng niềm đam mê

toán học cho các em học sinh.

Từ những lý do trên, đề tài đƣợc chọn là : “Dạy học bài toán chứng minh

bất đẳng thức ở trường trung học phổ thông theo phương pháp dạy học tích cực”

Page 6: Sinh viên Nguyễn Thị Lan Hƣơng · PDF fileLỜI CẢM ƠN Lời đầu tiên em xin bày tỏ lòng kính trọng và biết ơn sâu sắc đến Thầy giáo Thạc sĩ Nguyễn

KHÓA LUẬN TỐT NGHIỆP ĐẠI HỌC Nguyễn Thị Lan Hương – K36B SP Toán

2

2. Mục đích và nhiệm vụ nghiên cứu.

Mục đích nghiên cứu là vận dụng một số phƣơng pháp dạy học tích cực

trong dạy học bài toán chứng minh bất đẳng thức cho học sinh THPT.

Nhiệm vụ nghiên cứu:

- Nghiên cứu về phƣơng pháp dạy học gợi mở vấn đáp, phát hiện và giải

quyết vấn đề.

- Vận dụng các phƣơng pháp dạy học tích cực trong dạy học bài toán

chứng minh bất đẳng thức ở trƣờng THPT.

3. Đối tƣợng, phạm vi nghiên cứu.

Đối tƣợng nghiên cứu: Các bài toán chứng minh bất đẳng thức ở trƣờng

trung học phổ thông.

Phạm vi nghiên cứu: Một số lớp các bài toán thƣờng gặp về bất đẳng thức

ở trƣờng trung học phổ thông.

4. Phƣơng pháp nghiên cứu.

Nghiên cứu lý luận.

Tìm hiểu, nghiên cứu những vấn đề liên quan đến đề tài định hƣớng cho

việc nghiên cứu; phân tích và tổng hợp những quan điểm dựa trên các tài liệu về

tâm lý học, giáo dục học, phƣơng pháp dạy học môn Toán và các tài liệu về bất

đẳng thức.

5. Ý nghĩa của đề tài.

Góp phần đổi mới phƣơng pháp giảng dạy, phát huy tính tích cực, chủ

động, sáng tạo cho học sinh. Từ đó nâng cao chất lƣợng dạy và học bất đẳng thức

ở trƣờng THPT.

Page 7: Sinh viên Nguyễn Thị Lan Hƣơng · PDF fileLỜI CẢM ƠN Lời đầu tiên em xin bày tỏ lòng kính trọng và biết ơn sâu sắc đến Thầy giáo Thạc sĩ Nguyễn

KHÓA LUẬN TỐT NGHIỆP ĐẠI HỌC Nguyễn Thị Lan Hương – K36B SP Toán

3

CHƢƠNG 1. CƠ SỞ LÝ LUẬN VÀ THỰC TIỄN

1.1. Phƣơng pháp dạy học tích cực

1.1.1. Phƣơng pháp dạy học tích cực là gì?

a. Định hƣớng đổi mới phƣơng pháp dạy học

Định hƣớng đổi mới phƣơng pháp dạy và học đã đƣợc xác định trong Nghị

quyết Trung ƣơng 4 khóa VII (1 - 1993), Nghị quyết Trung ƣơng 2 khóa VIII (12

- 1996), đƣợc thể chế hóa trong Luật Giáo dục (12 - 1998), đƣợc cụ thể hóa trong

các chỉ thị của Bộ Giáo dục và Đào tạo, đặc biệt là chỉ thị số 15 (4 - 1999). Luật

Giáo dục, điều 24.2, đã ghi: "Phƣơng pháp giáo dục phổ thông phải phát huy tính

tích cực, tự giác, chủ động, sáng tạo của học sinh; phù hợp với đặc điểm của từng

lớp học, môn học; bồi dƣỡng phƣơng pháp tự học, rèn luyện kĩ năng vận dụng

kiến thức vào thực tiễn; tác động đến tình cảm, đem lại niềm vui, hứng thú học

tập cho học sinh".

Có thể nói cốt lõi của đổi mới dạy và học là hƣớng tới hoạt động học tập

chủ động, chống lại thói quen học tập thụ động.

b. Phƣơng pháp dạy học tích cực

Phƣơng pháp dạy học tích cực (PPDH tích cực) là một thuật ngữ rút gọn,

đƣợc dùng ở nhiều nƣớc để chỉ những phƣơng pháp giáo dục, dạy học theo

hƣớng phát huy tính tích cực, chủ động, sáng tạo của ngƣời học. "Tích cực" trong

PPDH - tích cực đƣợc dùng với nghĩa là hoạt động, chủ động, trái nghĩa với

không hoạt động, thụ động chứ không dùng theo nghĩa trái với tiêu cực.

PPDH tích cực hƣớng tới việc hoạt động hóa, tích cực hóa hoạt động nhận

thức của ngƣời học.

Muốn đổi mới cách học phải đổi mới cách dạy. Cách dạy chỉ đạo cách học,

nhƣng ngƣợc lại thói quen học tập của trò cũng ảnh hƣởng tới cách dạy của thầy.

Chẳng hạn, có trƣờng hợp học sinh đòi hỏi cách dạy tích cực hoạt động nhƣng

giáo viên chƣa đáp ứng đƣợc, hoặc có trƣờng hợp giáo viên hăng hái áp dụng

phƣơng pháp dạy học tích cực nhƣng không thành công vì học sinh chƣa thích

ứng, vẫn quen với lối học tập thụ động. Vì vậy, giáo viên phải kiên trì dùng cách

dạy hoạt động để dần dần xây dựng cho học sinh phƣơng pháp học tập chủ động

một cách vừa sức, từ thấp lên cao.Trong đổi mới phƣơng pháp dạy học phải có sự

hợp tác cả của thầy và trò, sự phối hợp nhịp nhàng hoạt động dạy với hoạt động

học thì mới thành công. Nhƣ vậy, việc dùng thuật ngữ "Dạy và học tích cực" để

phân biệt với "Dạy và học thụ động".

1.1.2. Đặc trƣng của các phƣơng pháp dạy học tích cực

a. Dạy và học thông qua tổ chức các hoạt động học tập của học sinh

Page 8: Sinh viên Nguyễn Thị Lan Hƣơng · PDF fileLỜI CẢM ƠN Lời đầu tiên em xin bày tỏ lòng kính trọng và biết ơn sâu sắc đến Thầy giáo Thạc sĩ Nguyễn

KHÓA LUẬN TỐT NGHIỆP ĐẠI HỌC Nguyễn Thị Lan Hương – K36B SP Toán

4

Trong phƣơng pháp dạy học tích cực, ngƣời học - đối tƣợng của hoạt động

"dạy", đồng thời là chủ thể của hoạt động "học" - đƣợc cuốn hút vào các hoạt

động học tập do GV tổ chức và chỉ đạo, thông qua đó tự lực khám phá những

điều mình chƣa rõ chứ không phải thụ động tiếp thu những tri thức đã đƣợc GV

sắp đặt. Đƣợc đặt vào những tình huống của đời sống thực tế, ngƣời học trực tiếp

quan sát, thảo luận, làm thí nghiệm, giải quyết vấn đề đặt ra theo cách suy nghĩ

của mình, từ đó nắm đƣợc kiến thức kĩ năng mới, vừa nắm đƣợc phƣơng pháp

"làm ra" kiến thức, kĩ năng đó, không rập theo những khuôn mâu sẵn có, đƣợc

bộc lộ và phát huy tiềm năng sáng tạo. Dạy theo cách này thì GV không chỉ giản

đơn truyền đạt tri thức mà còn hƣớng dẫn hành động. Chƣơng trình dạy học phải

giúp cho từng HS biết hành động và tích cực tham gia các chƣơng trình hành

động của cộng đồng.

b. Dạy và học chú trọng rèn luyện phƣơng pháp tự học

Phƣơng pháp tích cực xem việc rèn luyện phƣơng pháp học tập cho HS

không chỉ là một biện pháp nâng cao hiệu quả dạy học mà còn là một mục tiêu

dạy học. Phải quan tâm dạy cho HS phƣơng pháp học ngay từ bậc Tiểu học và

càng lên bậc học cao hơn càng phải đƣợc chú trọng. Trong các phƣơng pháp học

thì cốt lõi là phƣơng pháp tự học. Nếu rèn luyện cho ngƣời học có đƣợc phƣơng

pháp, kĩ năng, thói quen, ý chí tự học thì sẽ tạo cho họ lòng ham học, khơi dậy

nội lực vốn có trong mỗi con ngƣời, kết quả học tập sẽ đƣợc nhân lên gấp bội. Vì

vậy, ngày nay ngƣời ta nhấn mạnh mặt hoạt động học trong qúa trình dạy học, nỗ

lực tạo ra sự chuyển biến từ học tập thụ động sang tự học chủ động, đặt vấn đề

phát triển tự học ngay trong trƣờng phổ thông, không chỉ tự học ở nhà sau bài lên

lớp mà tự học cả trong tiết học có sự hƣớng dẫn của GV.

c. Dạy và học coi trọng hƣớng dẫn tìm tòi

Thông qua hƣớng dẫn tìm tòi, GV sẽ giúp các em phát triển kĩ năng giải

quyết vấn đề và khẳng định HS có thể xác định đƣợc phƣơng pháp học thông qua

hoạt động. Dấu hiệu đặc trƣng này không chỉ đặc biệt có hiệu quả với HS lớn

tuổi mà còn áp dụng đƣợc cho cả HS nhỏ tuổi nếu có tài liệu cụ thể và sự quan

tâm của GV. Kinh nghiệm cho thấy đây còn là cách để ngƣời học tìm lời giải đáp

cho các vấn đề đặt ra. Về phía ngƣời dạy cần có sự hƣớng dẫn kịp thời giúp cho

sự tìm tòi của ngƣời học đạt kết quả tốt.

d. Tăng cƣờng học tập cá thể, phối hợp với học tập hợp tác

Trong một lớp học mà trình độ kiến thức, tƣ duy của học sinh không thể

đồng đều tuyệt đối thì khi áp dụng phƣơng pháp tích cực buộc phải chấp nhận sự

phân hóa về cƣờng độ, tiến độ hoàn thành nhiệm vụ học tập, nhất là khi bài học

đƣợc thiết kế thành một chuỗi công tác độc lập. Áp dụng phƣơng pháp tích cực ở

trình độ càng cao thì sự phân hóa này càng lớn. Việc sử dụng các phƣơng tiện

Page 9: Sinh viên Nguyễn Thị Lan Hƣơng · PDF fileLỜI CẢM ƠN Lời đầu tiên em xin bày tỏ lòng kính trọng và biết ơn sâu sắc đến Thầy giáo Thạc sĩ Nguyễn

KHÓA LUẬN TỐT NGHIỆP ĐẠI HỌC Nguyễn Thị Lan Hương – K36B SP Toán

5

công nghệ thông tin trong nhà trƣờng sẽ đáp ứng yêu cầu cá thể hóa hoạt động

học tập theo nhu cầu và khả năng của mỗi học sinh. Tuy nhiên, trong học tập,

không phải mọi tri thức, kĩ năng, thái độ đều đƣợc hình thành bằng những hoạt

động độc lập cá nhân.

Trong nhà trƣờng, phƣơng pháp học tập hợp tác đƣợc tổ chức ở cấp nhóm,

tổ, lớp hoặc trƣờng. Đƣợc sử dụng phổ biến trong dạy học là hoạt động hợp tác

trong nhóm nhỏ 4 đến 6 ngƣời. Học tập hợp tác làm tăng hiệu quả học tập, nhất

là lúc phải giải quyết những vấn đề gay cấn, lúc xuất hiện thực sự nhu cầu phối

hợp giữa các cá nhân để hoàn thành nhiệm vụ chung.

Trong nền kinh tế thị trƣờng đã xuất hiện nhu cầu hợp tác xuyên quốc gia,

liên quốc gia, năng lực hợp tác phải trở thành một mục tiêu giáo dục mà nhà

trƣờng phải chuẩn bị cho HS.

e. Kết hợp đánh giá của thầy với tự đánh giá của trò

Trong dạy học, việc đánh giá HS không chỉ nhằm mục đích nhận định

thực trạng và điều chỉnh hoạt động học của trò mà còn đồng thời tạo điều kiện

nhận định thực trạng và điều chỉnh hoạt động dạy của thầy. Trong phƣơng pháp

tích cực, GV phải hƣớng dẫn học sinh phát triển kĩ năng tự đánh giá để tự điều

chỉnh cách học. GV cần tạo điều kiện thuận lợi để HS đƣợc tham gia đánh giá lẫn

nhau. Tự đánh giá đúng và điều chỉnh hoạt động kịp thời là năng lực rất cần cho

sự thành đạt trong cuộc sống mà nhà trƣờng phải trang bị cho HS. Theo hƣớng

phát triển các phƣơng pháp tích cực để đào tạo những con ngƣời năng động, sớm

thích nghi với đời sống xã hội, thì việc kiểm tra, đánh giá không thể dừng lại ở

yêu cầu tái hiện các kiến thức, lặp lại các kĩ năng đã học mà phải khuyến khích

trí thông minh, óc sáng tạo trong việc giải quyết những tình huống thực tế

Từ dạy và học thụ động sang dạy và học tích cực, GV không còn đóng vai

trò đơn thuần là ngƣời truyền đạt kiến thức, GV trở thành ngƣời thiết kế, tổ chức,

hướng dẫn các hoạt động độc lập hoặc theo nhóm nhỏ để HS tự lực chiếm lĩnh

nội dung học tập, chủ động đạt các mục tiêu kiến thức, kĩ năng, thái độ theo yêu

cầu của chƣơng trình. Trên lớp, HS hoạt động là chính, GV có vẻ nhàn nhã hơn

nhƣng trƣớc đó, khi soạn giáo án, GV đã phải đầu tƣ công sức, thời gian rất

nhiều so với kiểu dạy và học thụ động mới có thể thực hiện bài lên lớp với vai trò

là ngƣời gợi mở, xúc tác, động viên, cố vấn, trọng tài trong các hoạt động tìm tòi

hào hứng, tranh luận sôi nổi của HS. GV phải có trình độ chuyên môn sâu rộng,

có trình độ sƣ phạm lành nghề mới có thể tổ chức, hƣớng dẫn các hoạt động của

HS mà nhiều khi diễn biến ngoài tầm dự kiến của GV.

Page 10: Sinh viên Nguyễn Thị Lan Hƣơng · PDF fileLỜI CẢM ƠN Lời đầu tiên em xin bày tỏ lòng kính trọng và biết ơn sâu sắc đến Thầy giáo Thạc sĩ Nguyễn

KHÓA LUẬN TỐT NGHIỆP ĐẠI HỌC Nguyễn Thị Lan Hương – K36B SP Toán

6

Nhƣ vậy, PPDH tích cực chú trọng đến việc tích cực hóa hoạt động học

tập của HS, thông qua sự tổ chức hoạt động học của giáo viên cho HS. Yếu tố

quyết định đến sự thành công của dạy và học theo PPDH tích cực là GV phải

hƣớng dẫn HS có thể tự mình tìm tòi, khám phá ra tri thức, vận dụng và sáng tạo

để làm phong phú vốn tri thức của mình.

1.1.3. Các phƣơng pháp dạy học tích cực cần đƣợc phát huy ở trƣờng THPT

1.1.3.1. Phƣơng pháp gợi mở vấn đáp

a. Phƣơng pháp dạy học gợi mở - vấn đáp

Là quá trình tƣơng tác giữa GV và HS thông qua hệ thống các câu hỏi và

câu trả lời tƣơng ứng về một chủ đề nhất định đƣợc GV đặt ra. Qua việc trả lời hệ

thống câu hỏi dẫn dắt của GV, HS đƣợc thể hiện suy nghĩ, ý tƣởng của mình, từ

đó khám phá và lĩnh hội đối tƣợng học tập. GV giống nhƣ ngƣời tổ chức sự tìm

tòi, còn HS giống nhƣ ngƣời tự lực phát hiện kiến thức mới, HS có đƣợc niềm

vui của sự khám phá trƣởng thành thêm một bƣớc về trình độ tƣ duy.

b. Quy trình thực hiện

- Trƣớc giờ học

- Trong giờ học

- Sau giờ học

c. Ƣu điểm

- Kích thích tính tƣ duy độc lập của HS, dạy HS cách tự suy nghĩ đúng

đắn.

- Lôi cuốn HS tham gia học tập, tạo không khí lớp học sôi nổi, sinh động,

kích thích hứng thú học tập và lòng tự tin của HS.

- Tạo điều kiện HS giúp đỡ lẫn nhau trong học tập.

- GV thu nhận nhiều thông tin phản hồi từ phía ngƣời học.

d. Hạn chế: Khó soạn thảo, đòi hỏi GV phải chuẩn bị công phu nếu không kiến

thức sẽ thiểu tính hệ thống.

1.1.3.1. Phƣơng pháp dạy học phát hiện và giải quyết vấn đề

Phƣơng pháp dạy học mà ngƣời thầy tổ chức cho HS luôn đứng trƣớc tình

huống vấn đề về những nội dung toán học, tạo động lực cho HS tìm tòi, sáng tạo

những con đƣờng để giải quyết vấn đề đó. Từ đó, HS tìm ra và tích lũy tri thức

một cách tích cực, chủ động, sáng tạo. Vì tƣ duy chỉ xuất hiện khi gặp tình huống

có vấn đề (Rubinstein). Ngƣời GV giống nhƣ một đạo diễn vừa tạo ra tình huống

có vấn đề, vừa hƣớng dẫn HS chủ động giải quyết vấn đề.

Dạy học phát hiện và giải quyết vấn đề có các đặc điểm sau:

HS đƣợc đặt vào các tình huống có vấn đề chứ không phải đƣợc thông

báo tri thức dƣới dạng có sẵn.

Page 11: Sinh viên Nguyễn Thị Lan Hƣơng · PDF fileLỜI CẢM ƠN Lời đầu tiên em xin bày tỏ lòng kính trọng và biết ơn sâu sắc đến Thầy giáo Thạc sĩ Nguyễn

KHÓA LUẬN TỐT NGHIỆP ĐẠI HỌC Nguyễn Thị Lan Hương – K36B SP Toán

7

HS hoạt động tự giác, tích cực, chủ động, sáng tạo, tận lực huy động tri

thức và khả năng của mình để phát hiện và giải quyết vấn đề, chứ không phải

nghe thầy giảng một cách thụ động.

Mục đích của dạy học không chỉ làm cho HS lĩnh hội đƣợc kết quả của

quá trình phát hiện và giải quyết vấn đề, mà còn làm cho họ có khả năng tự tiến

hành quá trình nhƣ vậy.

Căn cứ vào mức độ độc lập của HS trong quá trình phát hiện và giải quyết

vấn đề, ngƣời ta chia thành các cấp độ sau:

+ Tự nghiên cứu vấn đề

+ Vấn đáp phát hiện và giải quyết vấn đề

+ Thuyết trình phát hiện và giải quyết vấn đề

Quy trình thực hiện:

B1: Phát hiện hoặc thâm nhập vấn đề

B2: Tìm giải pháp

B3: Trình bày giải pháp

B4: Nghiên cứu sâu giải pháp

1.2. Lý luận chung về bài toán

1.2.1. Bài toán và lời giải bài toán

1.2.1.1. Bài toán

Theo J.Pôlia: bài toán là việc đặt ra sự tìm kiếm có ý thức các phƣơng tiện

thích hợp để đạt mục đích xác định, nhiều khi trông thấy rõ ràngnhuwng không

đạt đƣợc ngay.

Hai yếu tố cấu thành lên bài toán:

- Mục đích xác định.

- Sự đòi hỏi thực hiện mục đích.

1.2.1.2. Lời giải

Lời giải bài toán là một tập hợp hữu hạn, sắp thứ tự các thao tác cần thiết để

đạt mục đích bài toán.

Ta đồng nhất các khái niệm bài giải, cách giải, đáp án đều theo nghĩa lời giải

ở trên.

Bài toán có thể có một lời giải, nhiều lời giải hoặc không có lời giải.

Giải đƣợc bài toán đƣợc hiểu là tìm ra và trình bày đúng ít nhất một lời giải

hoặc lý giải đƣợc rằng bài toán không giải đƣợc.

1.2.2. Phân loại giải bài toán và phƣơng pháp giải toán

1.2.2.1. Phân loại bài toán

Page 12: Sinh viên Nguyễn Thị Lan Hƣơng · PDF fileLỜI CẢM ƠN Lời đầu tiên em xin bày tỏ lòng kính trọng và biết ơn sâu sắc đến Thầy giáo Thạc sĩ Nguyễn

KHÓA LUẬN TỐT NGHIỆP ĐẠI HỌC Nguyễn Thị Lan Hương – K36B SP Toán

8

Có nhiều cách để phân loại các bài toán. Thông thƣờng phân loại bài toán

theo nhiều phƣơng pháp khác nhau để tiện mục đích sử dụng các bài toán đó.

a. Phân loại theo hình thức

Dựa vào kết luận của bài toán

- Toán chứng minh: Là những bài toán mà trong kết luận của nó đã thể

hiện rõ mục đích cuối cùng của mục đích bài toán.

- Toán tìm kiếm (toán tìm tòi): Bài toán mà trong kết luận của nó chƣa thể

hiện rõ kết luận cuối cùng của mục đích bài toán.

b. Phân loại theo phƣơng pháp giải toán

Dựa vào thuật toán chung để giải bài toán.

- Bài toán có angorit giải: Những bài toán có thuật toán chung để giải cho

lớp các bài toán chứa bài toán đó.

- Bài toán không có angorit giải: Những bài toán mà không có một thuật

toán chung nào để giải đƣợc lớp các bài toán chứa bài toán đã cho.

Chú ý: Số lƣợng các bài toán không có angorit giải là rất lớn so với các

bài toán có angorit giải.

c. Phân loại theo nội dung

Ngƣời ta căn cứ vào nội dung, lĩnh vực chuyên môn của bài toán để chia

bài toán thành lĩnh vực chuyên môn nhỏ hơn lĩnh vực chuyên môn ban đầu.

1.2.2. Phƣơng pháp giải bài toán

Bốn bƣớc giải bài toán của J.Pôlia (cho những bài toán không có angorit

giải)

Bƣớc 1: Tìm hiểu đề.

- Hãy xác định yếu tố đã cho và cần tìm (giả thiết, kết luận).

- Xác định yếu tố cố định, không đổi, thay đổi, biến thiên của bài toán.

- Cái đã cho có đủ để tìm cái cần tìm.

Bƣớc 2: Xây dựng chƣơng trình giải.

- Phƣơng pháp đi xuôi theo gợi ý của J.Pôlia là xuất phát từ những điều đã

cho (giả thiết) bằng suy luận và suy luận hợp logic từng bƣớc ta rút ra các kết

luận logic cho đến khi tìm đƣợc kết luận logic trùng với kết luận bài toán thì

dừng.

- Phƣơng pháp đi ngƣợc: Xuất phát từ kết luận bài toán (cái cần tìm) bằng

suy luận hợp logic từng bƣớc đi ngƣợc lên để tìm ra các tiền đề logic của chúng,

cho đến khi tìm đƣợc những tiền đề logic trùng với giả thiết (cái đã cho) thì

dừng.

- Phƣơng pháp sử dụng phép suy đoán: Để giải các bài toán tƣơng đối khó

(khi áp dụng phƣơng pháp đi xuôi hoặc đi ngƣợc bế tắc) thì ta thƣờng nghĩ tới

các bài toán có liên quan tƣơng tự đơn giản hơn đã biết cách giải.

Page 13: Sinh viên Nguyễn Thị Lan Hƣơng · PDF fileLỜI CẢM ƠN Lời đầu tiên em xin bày tỏ lòng kính trọng và biết ơn sâu sắc đến Thầy giáo Thạc sĩ Nguyễn

KHÓA LUẬN TỐT NGHIỆP ĐẠI HỌC Nguyễn Thị Lan Hương – K36B SP Toán

9

- Sử dụng phƣơng pháp, kết quả của các bài toán liên quan vào tìm lời giải

bài toán đã cho.

Bƣớc 3: Thực hiện chƣơng trình giải

Xuất phát từ điều đã cho hoặc đã biết nào đó bằng suy luận hợp logic ta rút

ra đƣợc các kết luận logic mới cho đến khi trùng kết luận bài toán thì dừng.

Bƣớc 4: Kiểm tra và khai thác bài toán

- Kiểm tra kết quả cuối cùng, các kết quả trung gian và toàn bộ lập luận của

lời giải bài toán.

- Khuyến khích học sinh tìm cách giải khác nếu có của bài toán.

- Nghiên cứu các bài toán có liên quan với bài toán đã cho: Bài toán ngƣợc,

bài toán khái quát hóa, bài toán tƣơng tự.

1.3. Những kiến thức cơ bản liên quan đến bất đẳng thức

1.3.1. Khái niệm, tính chất bất đẳng thức

a) Quan hệ thứ tự trong R

Trong tập hợp các số thực có quan hệ thứ tự, tức là :

Với mỗi cặp số thực ba, bất kì , luôn xảy ra một và chỉ một trong ba khả

năng:

- Hoặc a bằng b , ký hệu ba

- Hoặc a lớn hơn b , ký hiệu ba

- Hoặc a nhỏ hơn b , ký hiệu ba

b) Định nghĩa bất đẳng thức

Giả sử BA, là hai biểu thức (trƣờng hợp đặc biệt BA, có thể là hai số).

Mệnh đề “ A lớn hơn B ”, ký hiệu BA đƣợc gọi là một bất đẳng thức.

BA, gọi là các vế của bất đẳng thức ấy. Ngƣời ta cũng viết bất đẳng thức dƣới

dạng AB , đó là mệnh đề “ B nhỏ hơn A ” tƣơng đƣơng với mệnh đề trên.

Nhƣ bất cứ một mệnh đề toán học nào, bất đẳng thức BA có thể đúng

hoặc sai.

Quy ƣớc khi nói một bất đẳng thức mà không chỉ rõ gì hơn, thì ta hiểu rằng

đó là một bất đẳng thức đúng.

c) Bất đẳng thức suy rộng

Khi so sánh hai biểu thức A và B , nhiều khi chƣa thể kết luận dứt khoát:

A bằng B , A lớn hơn B , A nhỏ hơn B , mà chỉ có thể đƣa ra một kết luận mềm

dẻo hơn, chẳng hạn: A lớn hơn hoặc bằng B .

Do vậy, ngƣời ta sử dụng mệnh đề sau đây dƣới dạng ký hiệu:

BA : “ A lớn hơn hoặc bằng B ”.

BA : “ A nhỏ hơn hoặc bằng B ”.

Page 14: Sinh viên Nguyễn Thị Lan Hƣơng · PDF fileLỜI CẢM ƠN Lời đầu tiên em xin bày tỏ lòng kính trọng và biết ơn sâu sắc đến Thầy giáo Thạc sĩ Nguyễn

KHÓA LUẬN TỐT NGHIỆP ĐẠI HỌC Nguyễn Thị Lan Hương – K36B SP Toán

10

Các mệnh đề trên cũng đƣợc gọi là bất đẳng thức, rõ hơn: bất đẳng thức suy

rộng, để phân biệt với các bất đẳng thức nghiêm ngặt dạng BA , BA .

d) Tính chất bất đẳng thức

Tính chất 1: 0 baba .

Tính chất 2: cacb

ba

Tính chất 3: cbcaba .

Hệ quả 1: cbcaba .

Hệ quả 2: cbabca .

Tính chất 4: dbcadc

ba

.

Tính chất 5:

bcac

bcacba nếu

0

0

c

c

Hệ quả 1: baba .

Hệ quả 2:

)0(;

)0(;

cc

b

c

a

cc

b

c

a

ba

Tính chất 6: 0 baba

110 .

Tính chất 7: 0 ba *Nnba nn .

Tính chất 8: ba *1212 Nnba nn .

Tính chất 9: 0 ba *Nnba nn .

Tính chất 10: ba Nnba nn 1212 .

Ngoài ra ta cũng có các tính chất tương ứng với các bất đẳng thức suy

rộng.

1.3.2. Phân loại bài toán bất đẳng thức

Phân loại bài toán chứng minh bất đẳng thức theo phƣơng pháp giải.

Để chứng minh bất đẳng thức đại số, các phƣơng pháp phổ biến là:

PP1: Dùng phép biến đổi tƣơng đƣơng.

PP2: Phƣơng pháp phản chứng.

PP3: Dùng các tính chất cơ bản của bất đẳng thức.

PP4: Dùng bất đẳng thức tam giác.

PP5: Làm trội.

PP6: Quy nạp toán học.

PP7: Dùng bất đẳng thức Cauchy.

PP8: Dùng bất đẳng thức Bunhiacopski.

Page 15: Sinh viên Nguyễn Thị Lan Hƣơng · PDF fileLỜI CẢM ƠN Lời đầu tiên em xin bày tỏ lòng kính trọng và biết ơn sâu sắc đến Thầy giáo Thạc sĩ Nguyễn

KHÓA LUẬN TỐT NGHIỆP ĐẠI HỌC Nguyễn Thị Lan Hương – K36B SP Toán

11

PP9: Biến dạng của bất đẳng thức Bunhiacopski.

PP10: Dùng bất đẳng thức Bunhiacopski mở rộng.

PP11: Dùng bất đẳng thức Bernoulli.

PP12: Dùng tam thức bậc hai.

PP13: Phƣơng pháp lƣợng giác.

PP14: Dùng bất đẳng thức Jensen.

PP15: Dùng bất đẳng thức Tsebyshev.

PP16: Dùng hàm số.

PP17: Phƣơng pháp hình học.

1.3.3. Một số khó khăn và sai lầm thƣờng gặp của học sinh khi chứng minh

bất đẳng thức

- Học sinh sử dụng sai quy tắc suy luận trong chứng minh bất đẳng thức.

Ví dụ: Cho cba ,, dƣơng. Chứng minh rằng:

abcbacacbcba .

Hướng dẫn:

Ta thấy nhiều học sinh làm nhƣ sau:

Áp dụng bất đẳng thức 2

2

BAAB ta có

2

2

2))(( b

acbcbaacbcba

.

2

2

2c

bacacbbacacb

.

2

2

2a

cbabaccbabac

.

Nhân các vế tƣơng ứng ta có đpcm.

Các em đã quên điều kiện là khi nhân các bất đẳng thức cùng chiều này thì

biểu thức ở vế trái phải không âm. Nhƣ vậy ta phải xét hai trƣờng hợp:

Trƣờng hợp 1: cba , acb , bac không âm. Sử dụng kết quả

trên

Trƣờng hợp 2: Một trong ba đại lƣợng cba , acb , bac có ít

nhất một đại lƣợng âm. Khi đó có đúng một đại lƣơng âm vì tổng hai đại lƣợng

bất kì dƣơng. Vì vậy abcbacacbcba 0 .

- Học sinh quên điều kiện sử dụng bất đẳng thức.

Ví dụ : Chứng minh 4

11 aa .

Nhiều học sinh trình bày nhƣ sau:

Page 16: Sinh viên Nguyễn Thị Lan Hƣơng · PDF fileLỜI CẢM ƠN Lời đầu tiên em xin bày tỏ lòng kính trọng và biết ơn sâu sắc đến Thầy giáo Thạc sĩ Nguyễn

KHÓA LUẬN TỐT NGHIỆP ĐẠI HỌC Nguyễn Thị Lan Hương – K36B SP Toán

12

Áp dụng bất đẳng thức Côsi: 4

1)1(

2

1

2

11.

aa

aaaa .

Các em quên rằng không đủ điều kiện để khẳng định a và a1 không âm.

Ta phải trình bày nhƣ sau: 02

1

4

1)1(

2

aaa . Bất đẳng thức đúng.

Ngoài ra, học sinh có thể sai lầm khi sử dụng các phƣơng pháp của giải tích

nhƣ ngộ nhận: tích hai hàm đồng biến là hàm đồng biến, hiệu hai hàm nghịch

biến là hàm nghịch biến,...

Page 17: Sinh viên Nguyễn Thị Lan Hƣơng · PDF fileLỜI CẢM ƠN Lời đầu tiên em xin bày tỏ lòng kính trọng và biết ơn sâu sắc đến Thầy giáo Thạc sĩ Nguyễn

KHÓA LUẬN TỐT NGHIỆP ĐẠI HỌC Nguyễn Thị Lan Hương – K36B SP Toán

13

CHƢƠNG 2

VẬN DỤNG PHƢƠNG PHÁP DẠY HỌC TÍCH CỰC VÀO DẠY HỌC

BÀI TOÁN CHỨNG MINH BẤT ĐẲNG THỨC Ở TRƢỜNG THPT

2.1. Phƣơng pháp tích cực vận dụng trong chứng minh bất đẳng thức

Một nhà khoa học đã nói rằng một phát minh khoa học lớn cho phép giải

quyết một vấn đề lớn, nhƣng ngay cả trong việc giải một bài toán cũng có ít

nhiều phát minh. Bài toán mà bạn giải có thể là bình thƣờng nhƣng nếu nó khêu

gợi đƣợc trí tò mò và buộc bạn phải sáng tạo, và nếu tự mình giải lấy bài toán đó

thì bạn sẽ có thể biết đƣợc cái quyến rũ của sự sáng tạo cùng niềm vui thắng lợi.

Những tình cảm nhƣ vậy đến một độ tuổi nào đó, có thể khuấy động sự

ham thích công việc trí óc và mãi mãi để lại dấu vết trong cá tính ngƣời làm toán.

Khi HS đã có sự đam mê đối với toán học, lúc đó ngƣời thầy giáo hãy chỉ

cho HS một cách học hợp lý. Đứng trƣớc một bài toán, có phải sau khi tìm đƣợc

một lời giải đẹp, trình bày sạch sẽ là gấp sách lại hay không?

Trong dạy học môn Toán, GV thƣờng tạo ra các câu hỏi gợi mở để học sinh

phát hiện và giải quyết vấn đề, để tìm cách giải một bài toán (có thể theo bảng

gợi ý của Polya). Thậm chí, trong quá trình tìm lời giải một bài toán, học sinh có

khi tự đối thoại với chính mình. Các câu hỏi đƣợc lặp lại qua các bài bất đẳng

thức giúp học sinh tập luyện tri thức ăn khớp với tri thức phƣơng pháp. Bất đẳng

thức là một nội dung hay và khó. Nếu khả năng của học sinh còn hạn chế, ngƣời

thầy cần làm cho học sinh có cảm giác rằng tự HS làm đƣợc, do đó thầy phải

giúp đỡ kín đáo mà không bắt học sinh lệ thuộc vào mình. Ngƣời thầy phải đặt vị

trí mình là một học sinh, nghiên cứu trƣờng hợp cụ thể của HS, cố gắng hiểu xem

HS nghĩ gì, đặt ra câu hỏi để học sinh có thể tự mình trả lời đƣợc, bằng cách

ngƣời thầy sử dụng những kinh nghiệm của bản thân, nhớ lại những khó khăn và

những thành công của mình trong việc giải toán.

Trong dạy học môn Toán nói chung và dạy học bài tập chứng minh bất đẳng

thức nói riêng, yếu tố quyết định đến sự thành công của việc vận dụng phƣơng

pháp dạy học tích cực là hình thành cho học sinh kỹ năng phân tích, tìm ra đƣờng

lối chứng minh và khai thác đƣợc các bài toán chứng minh bất đẳng thức.

a) Phân tích tìm đƣờng lối chứng minh bài toán bất đẳng thức

Điều quan trọng trong dạy học một bài toán không chỉ là giúp HS tìm ra lời

giải bài toán đó, mà quan trong hơn cả là giúp cho HS biết cách tự mình tìm ra

lời giải cho các bài toán có liên quan, thuộc lớp bài toán đã cho. Để hình thành

cho HS kỹ năng phân tích, tìm ra đƣờng lối chứng minh cho một lớp bài toán thì

Page 18: Sinh viên Nguyễn Thị Lan Hƣơng · PDF fileLỜI CẢM ƠN Lời đầu tiên em xin bày tỏ lòng kính trọng và biết ơn sâu sắc đến Thầy giáo Thạc sĩ Nguyễn

KHÓA LUẬN TỐT NGHIỆP ĐẠI HỌC Nguyễn Thị Lan Hương – K36B SP Toán

14

GV cần hƣớng dẫn HS thực hành thông qua một số bài toán cụ thể thuộc lớp bài

toán đó.

Trƣớc hết, để chứng minh một bài toán bất đẳng thức nào đó, ta cần phải

hiểu đƣợc nội dung của nó. Cần tìm hiểu xem bài toán đã cho những gì? Ta cần

chứng minh điều gì? Với điều kiện đã cho thì ta có thể chứng minh đƣợc bất

đẳng thức đó không? Có gì mâu thuẫn không?

Tiếp theo, chúng ta hãy nhìn vào bất đẳng thức cần chứng minh. GV hƣớng

dẫn HS bằng cách nêu ra một số câu hỏi gợi mở. Bạn đã từng chứng minh bất

đẳng thức này chƣa? Đã từng chứng minh một bất đẳng thức nào ở dạng hơi

khác, nhƣng tƣơng tự nhƣ thế chƣa? Nhìn vào hai vế của bất đẳng thức, bạn có

nhận xét gì về vai trò của các số tham gia? Các biểu thức ở hai vế của bất đẳng

thức, cùng với mối liên hệ giữa các số tham gia, vai trò của chúng có gợi cho bạn

điều gì không? Chúng ta có thể áp dụng bất đẳng thức quen thuộc nào không? Có

thể vận dụng kết quả đã biết của bài toán nào đã chứng minh cho bài toán này

không? Có thể biến đổi, đặt ẩn phụ để đƣa bài toán cần chứng minh về bài toán

quen thuộc, dễ giải quyết hơn không? Có thể sử dụng các kiến thức liên quan đến

bất đẳng thức lƣợng giác, bất đẳng thức tam giác, bất đẳng thức hình học,… hay

không?...

Khi ngƣời thầy đặt câu hỏi cần nhằm vào hai mục đích: thứ nhất giúp học

sinh giải được một bài toán cụ thể, thứ hai là phát triển những khả năng của học

sinh để họ có thể tự lực giải những bài toán sau này. Hai mục đích này liên hệ

mật thiết với nhau. Nếu học sinh giải đƣợc bài toán cụ thể thì từ đó HS cũng có

thể có khả năng giải đƣợc bài toán tổng quát. Nhƣ vậy những câu hỏi mà thầy đặt

ra cho học sinh phải tổng quát và áp dụng vào nhiều trƣờng hợp. Nếu dùng nhiều

lần một câu hỏi, học sinh sẽ chú ý đến nó một cách trực giác và HS có thể tự đặt

ra đƣợc câu hỏi đó trong trƣờng hợp tƣơng tự. Nếu HS có thể tự đặt đƣợc câu hỏi

đó nhiều lần thì HS có thể rút ra đƣợc những ý kiến xác đáng. Ngƣời thầy phải

làm cho học sinh thấm nhuần những câu hỏi và những câu hỏi này sẽ góp phần

phát triển một thói quen của trí óc.

Gợi ý và câu hỏi là các cách GV đứng lớp giúp HS sử dụng vốn hiểu biết

có sẵn về một chủ đề. Gợi ý liên quan đến “các dấu hiệu” về những kinh nghiệm

có sẵn của học sinh. GV gợi ý cho HS, chờ đợi những kiến thức mới, điều này sẽ

khiến trong óc các em nảy ra những dự đoán về những thông tin mới. Việc đặt ra

các câu hỏi cũng có một chức năng nhƣ vậy. Khi dạy học, cần tập trung vào

những vấn đề quan trọng, trọng tâm chứ không phải là vào những gì bất thƣờng.

Khoảng thời gian “chờ đợi” trƣớc khi tiếp nhận nhận câu trả lời của HS có tác

dụng làm cho hiểu biết của các em sâu sắc hơn.

Page 19: Sinh viên Nguyễn Thị Lan Hƣơng · PDF fileLỜI CẢM ƠN Lời đầu tiên em xin bày tỏ lòng kính trọng và biết ơn sâu sắc đến Thầy giáo Thạc sĩ Nguyễn

KHÓA LUẬN TỐT NGHIỆP ĐẠI HỌC Nguyễn Thị Lan Hương – K36B SP Toán

15

Khi thầy hƣớng dẫn HS qua một hệ thống câu hỏi gợi mở, HS từng bƣớc

suy nghĩ trả lời, phát hiện và giải quyết vấn đề, tìm kiếm kiến thức mới. Qua đó

tƣ duy và một số phẩm chất đạo đức nảy nở và phát triển nhƣ tính chủ động, tự

tin, niềm phấn khởi, hứng thú dẫn đến tƣ duy sáng tạo trong việc chọn câu trả lời

chính xác. Tƣ duy và tính cách hầu nhƣ vô hình, khó thấy nhƣng lại thấm dần

vào trí tuệ, hình thành nên nhân cách ngƣời lao động sáng tạo sau này. Tƣ duy và

tính cách không hình thành theo kiểu kiến thức mà thấm dần theo kiểu “lắng

đọng phù sa”, mỗi ngày một ít rất khó thấy, tích luỹ lâu ngày mới thấy rõ, giống

nhƣ từng hạt cát nhỏ li ti coi nhƣ không đáng kể, lâu ngày tích lại thành bãi phù

sa. Một vài hạt cát nhỏ thì chẳng có ý nghĩa gì nhƣng bãi cát phù sa thì rất có ý

nghĩa.

Ví dụ: Cho cba ,, là các số dƣơng. Chứng minh rằng:

1888 222

abc

c

cab

b

bca

a.

Hướng dẫn

+ Bất đẳng thức này có tƣơng tự bất đẳng thức nào đã gặp không?

Đó là bất đẳng thức

czbyax

cba

z

c

y

b

x

a

2

+ Có thể sử dụng kết quả của nó để giải quyết bài toán không?

abcccabbbcaa

cba

abc

c

cab

b

bca

a

888888 222

2

222

+ Nhƣ vậy, để chứng minh bài toán, ta cần chứng minh điều gì?

Ta cần chứng minh:

1888 222

2

abcccabbbcaa

cba

Hay 2222 888 cbaabcccabbbcaa

+ Bất đẳng thức này có gần gũi với bất đẳng thức quen thuộc nào không?

Bất đẳng thức BunhiaCopski: 2222222zyxcbaczbyax

+ Có thể áp dụng đƣợc bất đẳng thức này không?

abccbacba

abccccabbbbcaaa

abcccabbbcaa

24

)8()8()8(

888

333

2222

2222

+ Nhƣ vậy, đến đây để chứng minh bất đẳng thức đã cho, ta cần chứng minh điều

gì?

Page 20: Sinh viên Nguyễn Thị Lan Hƣơng · PDF fileLỜI CẢM ƠN Lời đầu tiên em xin bày tỏ lòng kính trọng và biết ơn sâu sắc đến Thầy giáo Thạc sĩ Nguyễn

KHÓA LUẬN TỐT NGHIỆP ĐẠI HỌC Nguyễn Thị Lan Hương – K36B SP Toán

16

Ta cần chứng minh 3333 24 cbaabccba

+ Chứng minh bất đẳng thức trên nhƣ thế nào?

abccbaabccbaabccba

abccabcabcbacbacba

2433.3.3

33

3333 2223333

3333

Vậy bất đẳng thức đã cho đã đƣợc chứng minh.

Đẳng thức xảy ra cba .

b. Khai thác bài toán chứng minh bất đẳng thức

Hình thành cho HS kỹ năng giải quyết các bài toán chứng minh bất đẳng

thức là thành công bƣớc đầu trong việc vận dụng phƣơng pháp dạy học tích cực

vào dạy học bài toán chứng minh bất đẳng thức. Để đạt đƣợc hiệu quả cao hơn

nữa, GV còn cần phải hƣớng dẫn cho các em biết cách khai thác các bài toán

chứng minh bất đẳng thức, làm cho các em hiểu đƣợc ý nghĩa của việc làm này,

từ đó tự các em nảy sinh trong suy nghĩ sự ham thích, say mê nghiên cứu, tìm tòi,

khai thác sâu hơn các bài toán chứng minh bất đẳng thức. Khai thác các bài toán

chứng minh bất đẳng thức có thể là tìm thêm nhiều lời giải cho bài toán của

chúng ta, hoặc cũng có thể là từ bài toán ban đầu ta hãy sáng tạo thêm nhiều bài

toán mới, là trƣờng hợp đặc biệt, tƣơng tự, hay tổng quát hơn.

Ví dụ: Cho

1

0,,

zxyzxy

zyx

Chứng minh rằng 4

9

)(

1

)(

1

)(

1222

xzzyyx.

+ Xét 2

tan;2

tan;2

Atan

Cz

Byx với A, B, C là ba góc một tam giác.

Ta có

12

tan.2

tan2

tan.2

tan2

tan.2

tan

02

tan,2

tan,2

tan

ACCBBA

CBA

Khi đó

2cos

2cos.

2cos

2sin

2cos.

2cos

2tan

2

Atan

1

)(

1

2

22

2

22

22 C

BA

BA

BA

Byx

.

Từ đó ta có bài toán

Bài 1: Cho tam giác ABC. Chứng minh rằng

2cos

2cos.

2cos

2

22

C

BA

+

2cos

2cos.

2cos

2

22

A

CB

+4

9

2cos

2cos.

2cos

2

22

B

AC

.

Page 21: Sinh viên Nguyễn Thị Lan Hƣơng · PDF fileLỜI CẢM ƠN Lời đầu tiên em xin bày tỏ lòng kính trọng và biết ơn sâu sắc đến Thầy giáo Thạc sĩ Nguyễn

KHÓA LUẬN TỐT NGHIỆP ĐẠI HỌC Nguyễn Thị Lan Hương – K36B SP Toán

17

+ Xét CzByAx cot,cot,cot . Với A, B, C là các góc trong tam giác ABC.

Ta có:

1cot.cotcot.cotcot.cot

0cot,cot,cot

ACCBBA

CBA

Khi đó

C

BA

BAyx sin

sin.sin

cotcot

11

Từ đó ta có bài toán

Bài 2: Cho tam giác ABC. Chứng minh rằng

4

9

sin

sin.sin

sin

sin.sin

sin

sin.sin222

B

AC

A

CB

C

BA.

+ Áp dụng định lý sin ta có : 22

22

2

2

4

2

2.

2

sin

sin.sin

cR

ba

R

cR

b

R

a

C

BA

.

Từ đó ta có bài toán

Bài 3: Cho tam giác ABC. Chứng minh

2

2

22

2

22

2

22

9Rb

ac

a

cb

c

ba .

Nhận xét. Đây là một kết quả rất đẹp, khó nhận ra vì 22229 cbaR với

mọi tam giác.

Bất đẳng thức là một nội dung khó và phƣơng pháp giải rất đa dạng,

chính vì thế nó cũng làm cho nhiều GV phổ thông khó khăn trong việc dạy học

theo phƣơng pháp tích cực cho HS. Việc đƣa ra đƣợc hệ thống câu hỏi gợi mở,

vấn đáp, phát hiện và giải quyết vấn đề tác dụng tích cực đến tri giác, tƣ duy của

HS. Trí nhớ là hoạt động của phản xạ có điều kiện, thông tin cần lặp đi lặp lại

nhiều lần mới thành lập được phản xạ có điều kiện. Do đó đƣa ra một hệ thống

câu hỏi gợi mở, vấn đáp, từng bƣớc phát hiện và giải quyết vấn đề tạo cho HS

một thói quen tƣ duy tích cực, chủ động và lặp đi lặp lại theo từng dạng bài toán

là một phƣơng pháp hiệu quả giúp tăng cƣờng sức nhớ, rèn luyện kỹ năng giải

toán.

Điểm quan trọng của dạy học phát hiện và giải quyết vấn đề không phải là

những câu hỏi mà là tình huống gợi vấn đề. Trong nhiều trƣờng hợp, việc phát

hiện và giải quyết vấn đề của HS có thể diễn ra chủ yếu là nhờ tình huống gợi

vấn đề chứ không phải là nhờ những câu hỏi mà thầy đặt ra. Đứng trƣớc nhiều

Page 22: Sinh viên Nguyễn Thị Lan Hƣơng · PDF fileLỜI CẢM ƠN Lời đầu tiên em xin bày tỏ lòng kính trọng và biết ơn sâu sắc đến Thầy giáo Thạc sĩ Nguyễn

KHÓA LUẬN TỐT NGHIỆP ĐẠI HỌC Nguyễn Thị Lan Hương – K36B SP Toán

18

bài toán khó, nhiều khi HS không hoàn toàn phải sáng tạo cái mới mà phải biết

tổng hợp những kết quả đã học. Những câu hỏi của thầy giáo không hẳn là câu

hỏi gợi vấn đề mà có khi nhằm vào mục đích giúp tái hiện lại những kết quả đã

có, nhớ lại những bài toán phụ, những bổ đề áp dụng giải đƣợc bài toán trƣớc

mắt.

Sự khai thác hiệu quả các bài toán chứng minh bất đẳng thức làm cho các

em thấy đƣợc những vẻ đẹp của các bài toán này, càng thêm đam mê, yêu thích

hơn với các bài toán chứng minh bất đẳng thức. Khi xuất phát từ trong chính suy

nghĩ, ý thức của các em sự say mê, yêu thích thì việc học Toán nói chung và học

nội dung bất đẳng thức nói riêng trở lên dễ dàng, hiệu quả. Và khi ấy, chính sự

tích cực, chủ động và sáng tạo của các em trong việc học đã tạo lên thành công.

Thành công ở đây không chỉ là thành công trong một giờ dạy, trong một nội dung

bài học, mà điều quan trọng là đã rèn luyện cho các em tính tích cực, chủ động,

thói quen tƣ duy sáng tạo trong học tập, làm lên những thành công lớn trong việc

học của các em.

2.2. Vận dụng phƣơng pháp tích cực trong chứng minh bất đẳng thức

2.2.1. Phƣơng pháp chứng minh bất đẳng thức dựa vào các bất đẳng thức cơ

bản

2.2.1.1. Các bất đẳng thức cơ bản

a. Bất đẳng thức Cauchy cho n số không âm

* Dạng tổng quát: Gỉa sử naaa ,...,, 21 là n số thực không âm. Khi đó ta có:

+ Dạng 1: nn

n aaan

aaa...

...21

21

+ Dạng 2: nnn aaanaaa ....... 2121

+ Dạng 3:

n

n

n aaan

aaa...

...21

21

Đẳng thức xảy ra naaa ...21 0

* Hệ quả:

+ Nếu Saaa n ...21 là const thì ta có 1 2ax( ... )

n

n

SM a a a

n

xảy ra

khi và chỉ khi 1 2 ... n

Sa a a

n

+ Nếu Paaa n ...21 const thì ta có nn PnaaaMin .)...( 21 xảy ra khi

và chỉ khi 1 2 ... n

na a a P

b. Bất đẳng thức Bunhiacopxki

* Dạng tổng quát: Cho 2n số thực tùy ý nn bbbaaa ,...,,,,...,, 2121 , khi đó:

Page 23: Sinh viên Nguyễn Thị Lan Hƣơng · PDF fileLỜI CẢM ƠN Lời đầu tiên em xin bày tỏ lòng kính trọng và biết ơn sâu sắc đến Thầy giáo Thạc sĩ Nguyễn

KHÓA LUẬN TỐT NGHIỆP ĐẠI HỌC Nguyễn Thị Lan Hương – K36B SP Toán

19

+ Dạng 1: 22

2

2

1

22

2

2

1 ...... nn bbbaaa nnba...baba 2211

2

+ Dạng 2: nnnn babababbbaaa ......... 2211

22

2

2

1

22

2

2

1

+ Dạng 3: nnnn babababbbaaa ......... 2211

22

2

2

1

22

2

2

1

Dấu bằng ở dạng 1 và dang 2 xảy ra n

n

b

a

b

a

b

a ...

2

2

1

1

Dấu bằng ở dạng 3 xảy ra n

n

b

a

b

a

b

a ...

2

2

1

1 0

* Hệ quả:

+ Nếu cxaxaxa nn ...2211 là hằng số thì

n

n

n

na

x

a

x

a

x

aaa

cxxxMin

...

...)...(

2

2

1

1

22

2

2

1

222

2

2

1

+ Nếu 222

2

2

1 ... cxxx n là hằng số thì

nn xaxaxaMax ...2211 22

2

2

1 .... naaac 0...2

2

1

1 n

n

a

x

a

x

a

x

0..........2

2

1

122

2

2

12211 n

nnnn

a

x

a

x

a

xaaacxaxaxaMin

2.2.1.2. Các ví dụ minh họa

a. Sử dụng bất đẳng thức Cauchy

Ví dụ 1: Cho 3a . Tìm giá trị nhỏ nhất của biểu thức a

aS1

.

Hướng dẫn:

Sai lầm thƣờng gặp: 221

.21

MinSa

aa

aS

Nguyên nhân sai lầm: 11

2 a

aMinS mâu thuẫn với giả thiết 3a

Xét sự biến thiên của a

a1

, và a

aS1

để dự đoán 2MinS .

Ta có nhận xét rằng: Khi a càng tăng thì a

1càng giảm nhƣng độ tăng của a

rất lớn so với độ giảm của a

1 nên khi a tăng thì S càng lớn và từ đó dẫn đến dự

đoán khi 3a thì S nhận giá trị nhỏ nhất. Khi đó ta nói rằng 3

10MinS đạt tại

3a .

Do bất đẳng thức Cauchy xảy ra dấu bằng tại điều kện các số tham gia

phải bằng nhau, nên tại điểm 3a , ta không thể sử dụng bất đẳng thức Cauchy

Page 24: Sinh viên Nguyễn Thị Lan Hƣơng · PDF fileLỜI CẢM ƠN Lời đầu tiên em xin bày tỏ lòng kính trọng và biết ơn sâu sắc đến Thầy giáo Thạc sĩ Nguyễn

KHÓA LUẬN TỐT NGHIỆP ĐẠI HỌC Nguyễn Thị Lan Hương – K36B SP Toán

20

trực tiếp cho 2 số a và a

1 vì

3

13 . Lúc này ta giả sử dụng bất đẳng thức Cauchy

cho cặp số

a

a 1,

để tại 3a thì

a

a 1

tức là ta có sơ đồ:

93

3

1

3

11

3

3

a

a

a

Từ đó ta biến đổi S theo sơ đồ trên ta đƣợc

Lời giải đúng: 3

10

9

3.81.

92

9

81

9

1

a

aa

a

a

aaS . Với 3a thì

3

10MinS

Ví dụ 2: Cho 2a . Tìm giá trị nhỏ nhất của biểu thức 2

1

aaS

Hướng dẫn

Ta có nhận xét khi a tăng thì 2

1

a giảm, độ tăng của a lớn hơn so với độ

giảm của 2

1

a. Do đó a tăng thì S tăng, ta dự đoán S đạt giá trị nhỏ nhất khi

2a .

Ta có sơ đồ

82

4

1

4

11

2

2

2

a

a

a

Sai lầm thƣờng gặp:

4

9

4

7

4

2

8

2.7

2.8

2

8

7

8

2

8

71.

8.2

8

71

8

1222

a

a

a

a

aa

a

a

aaS

Với 2a thì 4

9MinS .

Nguyên nhân sai lầm: Mặc dù ta đã biến đổi S theo sơ đồ và 4

9MinS là

đáp số đúng nhƣng cách giải trên đã mắc sai lầm trong việc đánh giá mẫu số:

“Nếu 2a thì 4

2

2.8

2

8

2

a là đánh giá sai”

Để điều chỉnh lời giải trên thành lời giải đúng ta cần phải biến đổi S sao

cho khi sử dụng bất đẳng thức Cauchy sẽ khử hết biến số a ở mẫu số.

Lời giải đúng: Biến đổi S và sử dụng bất đẳng thức Cauchy ta có:

Page 25: Sinh viên Nguyễn Thị Lan Hƣơng · PDF fileLỜI CẢM ƠN Lời đầu tiên em xin bày tỏ lòng kính trọng và biết ơn sâu sắc đến Thầy giáo Thạc sĩ Nguyễn

KHÓA LUẬN TỐT NGHIỆP ĐẠI HỌC Nguyễn Thị Lan Hương – K36B SP Toán

21

4

9

8

2.61.

8.

8.3

8

61

88

13

222

a

aaa

a

aa

aaS .

Với 2a thì 4

9MinS .

Ví dụ 3: Cho 0, ba . Chứng minh rằng: 2

5

ba

ab

ab

ba

Hướng dẫn:

+ Do VT của bất đẳng thức cần chứng minh là biểu thức đối xứng với a và b

nên có thể dự đoán dấu đẳng thức xảy ra khi nào?

0 ba .

Ta có sơ đồ: 42

2

1

2

1

2

2

a

a

ba

ab

a

a

ab

ba

ba .

+ Dựa vào sơ đồ trên, hãy biến đổi VT của bất đẳng thức trên?

Ta có:

ab

ba

ba

ab

ab

ba

ba

ab

ab

ba

3

4

+ Áp dụng bất đẳng thức Cauchy cho 2 số dƣơng ba

ab

ab

ba

,

4, ta có:

2

53.

4.2

3

4

ab

ba

ba

ab

ab

ba

ab

ba

ba

ab

ab

ba

ba

ab

ab

ba.

Kết luận: Bất đẳng thức đƣợc chứng minh.

Đẳng thức xảy ra 0 ba .

Ví dụ 4: Cho

1

0,,

cba

cba. Chứng minh rằng: 333 accbbaS 3 18 .

Hướng dẫn:

Sai lầm thƣờng gặp

3

111.1.

3

111.1.

3

111.1.

33

33

33

acacac

cbcbcb

bababa

Cộng vế với vế các bất đẳng thức trên, ta đƣợc:

3

8

3

62

cbaS

Nguyên nhân sai lầm

Page 26: Sinh viên Nguyễn Thị Lan Hƣơng · PDF fileLỜI CẢM ƠN Lời đầu tiên em xin bày tỏ lòng kính trọng và biết ơn sâu sắc đến Thầy giáo Thạc sĩ Nguyễn

KHÓA LUẬN TỐT NGHIỆP ĐẠI HỌC Nguyễn Thị Lan Hương – K36B SP Toán

22

3232

1

1

1

3

8cba

ac

cb

ba

S vô lý.

+ Dự đoán đẳng thức xảy ra khi nào?

Vì S là một biểu thức đối xứng với cba ,, nên đẳng thức xảy ra tại điều

kiện

3

2

3

1

1

accbbacba

cba

cba

+ Trình bày lời giải:

Sử dụng biến đổi và bất đẳng thức Cauchy ta có:

3

3

2

3

2

.4

9

3

2.

3

2..

4

9

3

3

2

3

2

.4

9

3

2.

3

2..

4

9

3

3

2

3

2

.4

9

3

2.

3

2..

4

9

3333

3333

3333

ac

acac

cb

cbcb

ba

baba

Cộng vế với vế các bất đẳng thức trên, ta có:

333 183

6.

4

9

3

42.

4

9

cbaS

+ Kết luận: Bất đẳng thức đƣợc chứng minh. Đẳng thức xảy ra

3

1

3

2 cbaaccbba .

Ví dụ 5: Cho zyx ,, là các số dƣơng thỏa mãn 6 zyx .

Chứng minh rằng: 111 444888 zyxzyx .

Hướng dẫn:

Ta có nhận xét thông thƣờng các bài toán bất đẳng thức có giả thiết cho

các số tham gia là các số dƣơng và có tổng hoặc tích không đổi, ta thƣờng sẽ

nghĩ ngay đến việc sử dụng bất đẳng thức Cauchy.

Trong bài toán này ta cũng sẽ sử dụng bất đẳng thức Cauchy.

Vì vai trò của zyx ,, trong giả thiết và trong bất đẳng thức cần chứng minh

là nhƣ nhau. Dự đoán dấu bằng của bất đẳng thức xảy ra khi nào?

Dấu bằng của bất đẳng thức xảy ra 2 zyx .

Trình bày lời giải:

Áp dụng bất đẳng thức Cauchy, ta có:

Page 27: Sinh viên Nguyễn Thị Lan Hƣơng · PDF fileLỜI CẢM ƠN Lời đầu tiên em xin bày tỏ lòng kính trọng và biết ơn sâu sắc đến Thầy giáo Thạc sĩ Nguyễn

KHÓA LUẬN TỐT NGHIỆP ĐẠI HỌC Nguyễn Thị Lan Hương – K36B SP Toán

23

zzzzz

yyyyy

xxxxx

4.1264.8.8.36488

4.1264.8.8.36488

4.1264.8.8.36488

3

3

3

Cộng từng vế của các bất đẳng thức trên ta đƣợc

zyxzyx 4441264.38882

Áp dụng bất đẳng thức Cauchy cho ba số zyx 8,8,8 ta có:

1928.38.38.8.8.3888 3 633 zyxzyxzyx

111

111

444888

44434441219288828883

zyxzyx

zyxzyxzyxzyx

+ Kết luận: Bất đẳng thức đƣợc chứng minh.

Dấu đẳng thức xảy ra 2 zyx .

Ví dụ 6: Chứng minh rằng: 0,,,2

3

2

3

2

3

cbacbaa

c

c

b

b

a

Hướng dẫn:

Nhận xét cả hai vế là các biểu thức bậc nhất nên biểu thức cộng thêm cũng

là bậc nhất.

Sử dụng bất đẳng thức Cauchy ta có:

cbaa

c

c

b

b

a

cbabbb

abb

b

acba

a

c

c

b

b

a

cyccyc

2

3

2

3

2

3

32

3

2

3

2

3

2

3

2

3

3...32

Dấu bằng xảy ra 0 cba .

Ví dụ 7: Chứng minh rằng: 0,,,333

cbacabcaba

c

c

b

b

a

Hướng dẫn:

Ta có nhận xét cả hai vế của bất đẳng thức là các biểu thức bậc hai nên

biểu thức cộng thêm cũng là bậc hai.

Sử dụng bất đẳng thức Cauchy ta có:

cyccyc

cabcabbcc

b

b

abc

c

b

b

acabcab

a

c

c

b

b

a3...32 3

3333333

cabcaba

c

c

b

b

a

333

. Dấu bằng xảy ra 0 cba

Ví dụ 8: Cho

1

0,

ba

ba. Chứng minh rằng: 6

1122

abba

S

Hướng dẫn:

Page 28: Sinh viên Nguyễn Thị Lan Hƣơng · PDF fileLỜI CẢM ƠN Lời đầu tiên em xin bày tỏ lòng kính trọng và biết ơn sâu sắc đến Thầy giáo Thạc sĩ Nguyễn

KHÓA LUẬN TỐT NGHIỆP ĐẠI HỌC Nguyễn Thị Lan Hương – K36B SP Toán

24

Ta có nhận xét vế trái của bất đẳng thức là tổng của hai phân số cùng bậc,

thông thƣờng khi gặp bài toán có dạng nhƣ trên ta sử dụng đánh giá bởi dạng

cộng mẫu số.

Sử dụng “Dạng cộng mẫu số của bất đẳng thức Cauchy:

Cho 0,...,, 21 nxxx với Nnn ,2 . Khi đó ta có:

nn xxx

n

xxx

...

1...

11

21

2

21

Ta có:

6

624

2

1

2

4

2

1

2

112222222

bababaababbaababbaS

Bất đẳng thức đƣợc chứng minh.

Đẳng thức xảy ra 2

1 ba .

b. Sử dụng bất đẳng thức BunhiaCopski

Ví dụ 1: Cho cba ,, là độ dài ba cạnh của tam giác. Chứng minh rằng:

cbacba

c

bac

b

acb

a

222

.

Hướng dẫn:

Chúng ta sẽ sử dụng “Dạng cộng mẫu số của bất đẳng thức BunhiaCopski:

n

n

x

a

x

a

x

a2

2

2

2

1

2

1 ...

0,...,,,...

...21

21

2

21

n

n

n xxxxxx

aaa”

+ Hãy trình bày lời giải?

Ta có:

cba

cbabacacb

cba

cba

c

bac

b

acb

a

2222

+ Kết luận: Bất đẳng thức đƣợc chứng minh.

Đẳng thức xảy ra 0 cba

Ví dụ 2: Cho 0,, cba . Chứng minh:

3

222 222

2

222

2

222

2

bac

ac

acb

cb

cba

ba

Hướng dẫn:

+ Sử dụng dạng cộng mẫu số cho từng phân số trong tổng ở vế trái của bất đẳng

thức?

Ta có:

Page 29: Sinh viên Nguyễn Thị Lan Hƣơng · PDF fileLỜI CẢM ƠN Lời đầu tiên em xin bày tỏ lòng kính trọng và biết ơn sâu sắc đến Thầy giáo Thạc sĩ Nguyễn

KHÓA LUẬN TỐT NGHIỆP ĐẠI HỌC Nguyễn Thị Lan Hương – K36B SP Toán

25

22

2

22

2

2222

2

222

2

22

2

22

2

2222

2

222

2

22

2

22

2

2222

2

222

2

2

2

2

ba

a

bc

c

babc

ac

bac

ac

ac

c

ab

b

acab

cb

acb

cb

cb

b

ca

a

cbca

ba

cba

ba

Cộng vế với vế các bất đẳng thức trên, ta có:

322

22

22

22

22

22

ac

ac

cb

cb

ba

baVT .

Kết luận: Bất đẳng thức đƣợc chứng minh.

Đẳng thức xảy ra 0 cba .

Ví dụ 3: Cho

2

3

0,,

cba

cba. Tìm Min của: S

2

2 1

ba +

2

2 1

cb +

2

2 1

ac .

Hướng dẫn:

Xét bất đẳng thức quen thuộc BunhiaCopski:

2211

2

2

2

1

2

2

2

1 bababbaa . Dấu bằng xảy ra 02

2

1

1 b

a

b

a.

Ý nghĩa: Chuyển đổi một biểu thức ở trong căn bậc hai thành một biểu

thức linh động hơn ở ngoài căn thức. Xét đánh giá giả định với các số , .

ac

ac

ac

cb

cb

cb

ba

ba

ba

22

22

2

2

222

2

22

22

2

2

222

2

22

22

2

2

222

2

1111

1111

1111

Cộng vế với vế các bất đẳng thức trên, ta có:

022

1111S

cbacbaS

Do S là một biểu thức đối xứng với cba ,, nên dự đoán 0SS tại

2

1 cba , khi đó tất cả các bất đẳng thức trong tổng trên đồng thời phải xảy

ra dấu bằng tức là ta có sơ đồ:

Page 30: Sinh viên Nguyễn Thị Lan Hƣơng · PDF fileLỜI CẢM ƠN Lời đầu tiên em xin bày tỏ lòng kính trọng và biết ơn sâu sắc đến Thầy giáo Thạc sĩ Nguyễn

KHÓA LUẬN TỐT NGHIỆP ĐẠI HỌC Nguyễn Thị Lan Hương – K36B SP Toán

26

4

1

1

4

111

1

1

1

2

a

c

c

b

b

a

a

c

c

b

b

a

cba

Ta có lời giải sau:

ac

ac

ac

cb

cb

cb

ba

ba

ba

4

17

141

1

17

11

4

17

141

1

17

11

4

17

141

1

17

11

22

2

2

2

2

22

2

2

2

2

22

2

2

2

2

Cộng vế với vế các bất đẳng thức trên, ta có:

.2

1732.

4

453

17

1

3

1.

4

453

17

1

1.

4

453

17

11.

1.

13.

4

15

4

1

4

1.

4

1..6

17

1

111

4

15

4

1

4

1

4

1

17

1444

17

1

336

cba

abccbacbaabc

cbacbacba

cbacbaS

Với 2 cba thì 2

173MinS .

Ví dụ 4: Cho 0,, cba thỏa mãn 102 abccba . Chứng minh rằng

6642

98

42

98

42

98 222

2

222

2

222

2

cba

c

bac

b

acb

aS

Hướng dẫn:

+ Hãy dự đoán đẳng thức xảy ra khi nào?

Đẳng thức xảy ra 2 cba .

+ Hãy tìm cách để chuyển biểu thức trong căn thành biểu thức bên ngoài căn linh

động hơn?

Áp dụng bất đẳng thức BunhiaCopski ta có:

Page 31: Sinh viên Nguyễn Thị Lan Hƣơng · PDF fileLỜI CẢM ƠN Lời đầu tiên em xin bày tỏ lòng kính trọng và biết ơn sâu sắc đến Thầy giáo Thạc sĩ Nguyễn

KHÓA LUẬN TỐT NGHIỆP ĐẠI HỌC Nguyễn Thị Lan Hương – K36B SP Toán

27

bcac

cba

c

abcb

bac

b

caba

acb

a

94

42

98.4182

94

42

98.4182

94

42

98.4182

222

2

222

2

222

2

Cộng vế với vế các bất đẳng thức trên, ta đƣợc:

6624

727210.6122612

6222.4

2.4

2.4

2

622444

9111

4.24

Sabccba

cbaabcabcabccc

bb

aa

cbaabcbcacc

bb

aa

cabcabcbacba

S

Ví dụ 5: Cho ba, là hai số bất kỳ và yx, là hai số dƣơng. Chứng minh rằng:

yx

ba

y

b

x

a

222

(*)

Hướng dẫn:

+ Bất đẳng thức trên gợi cho ta suy nghĩ đến bất đẳng thức quen thuộc nào?

Bất đẳng thức BunhiaCopski 22222 bdacdcba

+ Hãy sử dụng bất đẳng thức BunhiaCopski chứng minh bất đẳng thức (*)?

Ta có:

222

bayxy

b

x

a yx

ba

y

b

x

a

222

(đpcm).

+ Đẳng thức xảy ra khi nào?

Khi y

b

x

a .

Ví dụ 6: Cho

18

3,2,1

zyx

zyx.

Chứng minh rằng 23321 444 zyxS .

Hướng dẫn:

+ Hãy áp dụng bất đẳng thức BunhiaCopski?

Ta có:

2312.3332133

3211113.12.11.1 222444

zyx

zyxzyxS

Kết luận: Bất đẳng thức đƣợc chứng minh.

Page 32: Sinh viên Nguyễn Thị Lan Hƣơng · PDF fileLỜI CẢM ƠN Lời đầu tiên em xin bày tỏ lòng kính trọng và biết ơn sâu sắc đến Thầy giáo Thạc sĩ Nguyễn

KHÓA LUẬN TỐT NGHIỆP ĐẠI HỌC Nguyễn Thị Lan Hương – K36B SP Toán

28

Đẳng thức xảy ra khi 5x , 6y , 7z .

Các bài tập luyện tập

Bài 1: Cho

2

3

0,,

cba

cba

. Chứng minh rằng cba

cbaS111

2

15 .

Bài 2: Cho

3

0,,

cba

cba.

Chứng minh rằng: 3333 33222 bacacbcba .

Bài 3: Chứng minh rằng: 0,,,222

3

5

3

5

3

5

cbacbaa

c

c

b

b

a.

Bài 4: Cho 0,, cba thỏa mãn 1 cba . Chứng minh rằng:

c

c

b

b

a

a

a

c

c

b

b

a

1

1

1

1

1

12 .

Bài 5: Chứng minh rằng:

0,,,3

133

3

33

3

33

3

zyxxzz

z

zyy

y

yxx

x.

2.2.2. Phƣơng pháp chứng minh bất đẳng thức dựa vào hàm số

2.2.2.1. Các kiến thức hàm số dùng trong chứng minh bất đẳng thức

a. Tính chất hàm số bậc nhất

Cho hàm số y f x ax b . Nếu xét trên đoạn ; thì đồ thị của nó là

một đoạn thẳng có hai đầu mút là hai điểm ;A f và ;B f . Do đó,

00, ;

0

ff x x

f

.

00, ;

0

ff x x

f

.

;min min ;f x f f

.

;max max ;f x f f

.

b. Tính chất tiếp tuyến của hàm số tại một điểm

Cho hàm số y f (x) . Tiếp tuyến tại 0 0 0M (x ;y ) thuộc vào đồ thị hàm số

có phƣơng trình:

0 0 0y y f '(x )(x x )

Chú ý:

+ Nếu hàm số y f (x) là lồi trên ba; và 0x (a;b) thì ta có:

0 0 0f (x) f '(x )(x x ) y .

Page 33: Sinh viên Nguyễn Thị Lan Hƣơng · PDF fileLỜI CẢM ƠN Lời đầu tiên em xin bày tỏ lòng kính trọng và biết ơn sâu sắc đến Thầy giáo Thạc sĩ Nguyễn

KHÓA LUẬN TỐT NGHIỆP ĐẠI HỌC Nguyễn Thị Lan Hương – K36B SP Toán

29

+ Nếu hàm số y f (x) là lõm trên ba; và 0x (a;b) thì ta có:

0 0 0f (x) f '(x )(x x ) y

Tuy nhiên các bất đẳng thức trên không có trong chƣơng trình nên khi sử

dụng học sinh cần phải chứng minh.

c. Đạo hàm của hàm số

Cho hàm số y f (x) xác định và liên tục trên D và 0x D . Hàm số đạt

cực trị tại 0x khi 0f '(x ) 0 (hoặc không xác định tại x0) và f’(x) đổi dấu khi qua

x0.

2.2.2.2. Một số ví dụ minh họa

a. Một số ví dụ minh họa sử dụng tính chất hàm số bậc nhất

Ví dụ 1: Cho , , 0;2a b c . Chứng minh 2 4a b c ab bc ca .

Hướng dẫn

Ta thấy vai trò của cba ,, là nhƣ nhau, cùng bậc nhất.

+ Với bài này ta có thể quy về xét hàm số bậc nhất không, sẽ xét hàm số bậc nhất

nào?

Coi một số làm ẩn (giả sử a ), hai số còn lại là tham số. Ta xét hàm số sau:

422 bccbacbaf

+ Chứng minh bất đẳng thức ban đầu, ta cần chứng minh điều gì?

Ta cần chứng minh

2 2 4 0,f a b c a b c bc 0;2a , , 0;2b c .

0 0, , 0;2

2 0

fb c

f

.

+ Mà 0 2 2 0f b c ; 2 0f bc , , 0;2b c .

Do đó, 0, , , 0;2f a a b c đpcm.

Ví dụ 2: Cho , , 0a b c thỏa mãn 1a b c .

Chứng minh 7

227

ab bc ca abc .

Hướng dẫn

Bài toán trên đã xuất hiện nhiều trong các tài liệu và đều đƣợc giải

theo phƣơng pháp ứng dụng đạo hàm.

Vì vai trò bình đẳng ngang nhau của các biến , ,a b c nên việc chọn phần tử

nhỏ nhất (lớn nhất) có thể làm cho giả thiết của bài toán đƣợc sáng tỏ thêm hay

nhƣ đƣợc cho thêm giả thiết. Tuy nhiên, việc đánh giá để qui về một biến cũng

không hề đơn giản. Ta xem lại cách giải sau:

Cách 1.

Page 34: Sinh viên Nguyễn Thị Lan Hƣơng · PDF fileLỜI CẢM ƠN Lời đầu tiên em xin bày tỏ lòng kính trọng và biết ơn sâu sắc đến Thầy giáo Thạc sĩ Nguyễn

KHÓA LUẬN TỐT NGHIỆP ĐẠI HỌC Nguyễn Thị Lan Hương – K36B SP Toán

30

+ Vì vai trò của , ,a b c bình đẳng nên ta luôn có thể giả sử

1

min , , 03

a a b c a .

+ Ta có 2 1 2VT ab ca bc abc a b c bc a

2

1 1 22

b ca a a

(vì 1 2 0a )

2

11 1 2

2

aa a a

+ Xét hàm 2

11 1 2

2

af a a a a

, với

10;

3a

. Lập bảng biến thiên,

tìm GTLN của hàm f a trên 1

0;3

, ta có điều phải chứng minh.

Cách 2.

+ Để chứng minh bất đẳng thức ban đầu, ta cần chứng minh điều gì?

Ta cần chứng minh

7

2 0, 0;1 , , 0, 127

f a b c bc a bc a b c a b c

70 , , 0, 1

27

71 0, , 0, 0

27

f bc b c b c

f b c bc b c b c

.

, 0, 1b c b c , 2

7 7 1 70 0

27 2 27 4 27

b cf bc

.

Ta có điều phải chứng minh.

Trong cách giải trên, khi quan sát bất đẳng thức cần chứng minh ta

thấy có thể qui về hàm số bậc cao nhất là 1, với biến a , tham số ,b c . Khi đó, bài

toán có hai tham số, mà việc khai thác điều kiện cho hai tham số trong từng

trƣờng hợp tại 0a , hoặc tại 1a không phải học sinh nào cũng phát hiện đƣợc.

Do đó, ta có thể hƣớng dẫn học sinh sử dụng những biến đổi đại số cơ bản để đƣa

về hàm số chỉ còn chứa một tham số nhƣ Cách 3 sau đây:

Cách 3.

+ Biến đổi bất đẳng thức cần chứng minh về dạng

7

1 2 027

a b c bc a 7

1 1 2 027

a a bc a .

+ Đặt bc=u ta có thể xét hàm số bậc nhất nào ?

Page 35: Sinh viên Nguyễn Thị Lan Hƣơng · PDF fileLỜI CẢM ƠN Lời đầu tiên em xin bày tỏ lòng kính trọng và biết ơn sâu sắc đến Thầy giáo Thạc sĩ Nguyễn

KHÓA LUẬN TỐT NGHIỆP ĐẠI HỌC Nguyễn Thị Lan Hương – K36B SP Toán

31

+ Yêu cầu của bài toán 7

1 1 2 027

f u a a u a

với

221

02 4

ab cu bc

, 0;1a .

2

0 0

, 0;110

4

f

aaf

.

+ Ta có

2

17 7 1 70 1 0

27 2 27 4 27

a af a a

, 0;1a

2

10

4

af

, 0;1a .

Ví dụ 3: Cho , , 0a b c thỏa mãn 3a b c . Chứng minh 2 2 2 4a b c abc .

Hướng dẫn

Cách 1.

+ Không mất tính tổng quát, ta giả sử min , , 0 1a a b c a .

+ Khi đó, 22 2 2 2 2a b c abc a b c bc a

2

22 3 22

b ca a a

(vì 2 0a )

2

22 33 2

2

aa a a

+ Ta đi tìm GTNN của hàm số 2

22 33 2

2

af a a a a

trên 0;1

+ Ta nhận thấy rằng bài toán có thể giải theo cách trên nhƣng lời giải chƣa thật

ngắn gọn, ta cùng tìm hiểu cách giải thứ 2 ngắn gọn và hiệu quả hơn dựa vào

hàm số bậc nhất.

Cách 2.

Ta sử dụng tính chất hàm số bậc nhất.

+ Hãy tìm hàm số bậc nhất và xét hàm số đó?

Biến đổi bất đẳng thức cần chứng minh về dạng

22 2 2 24 2 4a b c abc a b c bc abc

22 3 2 4 0a a bc a

+ Để chứng minh bất đẳng thức ban đầu, ta cần chứng minh điều gì?

Ta cần chứng minh 22 2 6 5 0f u a u a a

với 2

210 3

2 4

b cu bc a

, 0;3a

Page 36: Sinh viên Nguyễn Thị Lan Hƣơng · PDF fileLỜI CẢM ƠN Lời đầu tiên em xin bày tỏ lòng kính trọng và biết ơn sâu sắc đến Thầy giáo Thạc sĩ Nguyễn

KHÓA LUẬN TỐT NGHIỆP ĐẠI HỌC Nguyễn Thị Lan Hương – K36B SP Toán

32

2

0 0

,13 0

4

f

f a

0;3a

+ Mà 2

3 10 2 0

2 5f a

;

2 21 13 1 2 0

4 4f a a a

, 0;3a .

Do đó, 21

0, 0; 34

f u u a

điều phải chứng minh.

Ví dụ 4: Cho , , 0a b c thỏa mãn 1a b c . Chứng minh 3 3 3 16

4a b c abc .

Hướng dẫn

Cách 1.

+ Vì vai trò của , ,a b c bình đẳng nên ta luôn có thể giả sử

1

min , , 03

a a b c a

+ Khi đó, 33 3 3 36 3 6a b c abc a b c bc b c abc

33 1 3 3 1a a bc a

2

33 1 3. 3 12

b ca a a

(vì 3 1 0a )

2

33 11 3 3 1

2

aa a a

+ Xét hàm số 2

33 11 3 3 1

2

af a a a a

, với 1

0;3

a

. Tìm GTNN của

hàm f a trên 10;

3

ta có điều phải chứng minh.

Cách 2.

Sử dụng tính chất hàm số bậc nhất.

+ Dựa vào mối quan hệ giữa cba ,, và các phép biến đổi đại số hãy biến đổi vế

trái về dạng một hàm số bậc nhất?

+ Biến đổi bất đẳng thức cần chứng minh về dạng

3 3 1

3 64

a b ab a b c abc

3 3 1

1 3 1 64

c ab c c abc 2 39 3 3 3 0

4ab c c c

+ Đặt uab ta có hàm số bậc nhất nào ?

Ta có hàm số bậc nhất 4

33339 2 ccucuf

Page 37: Sinh viên Nguyễn Thị Lan Hƣơng · PDF fileLỜI CẢM ƠN Lời đầu tiên em xin bày tỏ lòng kính trọng và biết ơn sâu sắc đến Thầy giáo Thạc sĩ Nguyễn

KHÓA LUẬN TỐT NGHIỆP ĐẠI HỌC Nguyễn Thị Lan Hương – K36B SP Toán

33

+ Yêu cầu của bài toán 2 39 3 3 3 0,

4f u c u c c

với

22

10 , 0;1

2 4

ca bu ab c

+ Ta có 2

10 3 0

2f c

;

2

21 3

3 3 1 04 4

cf c c c

, 0;1c đpcm.

Ví dụ 5: Cho , , , 0;1a b c d . Chứng minh

1 1 1 1 1a b c d a b c d .

Hướng dẫn

+ Nhận xét thấy rằng vai trò của dcba ,,, là nhƣ nhau.

Hãy biến đổi vế trái của bất đẳng thức để đƣa về dạng hàm số bậc nhất

theo biến a ?

11111111 dcbdcbadcbaf .

+ Yêu cầu bài toán tƣơng đƣơng

1 1 1 1 1 1 1 1 0, 0;1f a b c d a b c d b c d a ,

, , 0;1b c d . (*).

+ Theo tính chất hàm số bậc nhất, điều kiện cần và đủ để (*) xảy ra là gì?

Điều kiện cần và đủ để (*) xảy ra là

0 0, , , 0;1

1 0

fb c d

f

.

+ Mà 1 0, , , 0;1f b c d b c d .

+ Ta cần chứng minh 0 1 1 1 1 0, , , 0;1f b c d b c d b c d .

+ Ta xét bài toán:

Chứng minh 1 1 1 1 0, , , 0;1b c d b c d b c d (1).

Ta thấy bài toán (1) tƣơng tự bài toán ban đầu, tuy nhiên đã giảm

bớt đi một biến. Do đó, tiếp tục cách làm trên ta có thể giải quyết triệt để đƣợc

bài toán ban đầu.

Yêu cầu bài toán (1)

1 1 1 1 1 1 0, 0;1 , , 0;1g b c d b c d c d b c d

0 0, , 0;1

1 0

gc d

g

+ Mà

1 0; 0 1 1 1 0g c d g c d c d cd 0 , , 0;1g b b c d

Page 38: Sinh viên Nguyễn Thị Lan Hƣơng · PDF fileLỜI CẢM ƠN Lời đầu tiên em xin bày tỏ lòng kính trọng và biết ơn sâu sắc đến Thầy giáo Thạc sĩ Nguyễn

KHÓA LUẬN TỐT NGHIỆP ĐẠI HỌC Nguyễn Thị Lan Hương – K36B SP Toán

34

Ta có điều phải chứng minh.

Một số bài toán, không phải lúc nào cũng có sẵn dạng hàm số

y ax b . Tuy nhiên, trong một số trường hợp, nhờ những biến đổi, đánh giá bất

đẳng thức đại số thích hợp ta có thể áp dụng tính chất hàm số y ax b trong

chứng minh. Ta xét các ví dụ sau:

Ví dụ 6: Chứng minh rằng

1 1 1 1, , , 0;11 1 1

a b ca b c a b c

b c c a a b

Hướng dẫn

+ Không mất tính tổng quát ta giả sử max , ,a a b c . Khi đó

1 1 11

a b cVT a b c

b c

+ Nhƣ vậy để chứng ming bất đẳng thức ban đầu, ta chỉ cần chứng minh bất đẳng

thức nào ?

+Ta cần chứng minh

1;0,,1;0,11111

cbacba

cb

cbaaf

Hay 1 1 1 1 0, 0;1 , , 0;11

a b cf a a b c a b c

b c

(6)

+ Theo tính chất hàm số bậc nhất điều kiện cần và đủ để có (6) là gì ?

Điều kiện là

01

00

f

f , ta cần chứng minh điều này là đúng.

+ Thật vậy, , 0;1b c , ta có

2 2

2 2 12

0 01 1 1

b cb c bc

bc b c b c bcb cf bc b c

b c b c b c

1 0f

Ta có điều phải chứng minh.

Ví dụ 7: Cho , , 0;1a b c . Chứng minh 2 2 2 2 2 2 1a b c a b b c c a .

Hướng dẫn

+ Biến đổi bất đẳng thức về dạng 2 2 2 2 21 1 0b a c a b c b c

+ Vì 0;1a nên 2a a , suy ra

2 2 2 2 2 2 2 2 2 2 21 1 1 1b a c a b c b c b a c a b c b c .

+ Ta đi chứng minh 2 2 2 2 2 21 1 0,b a c a b c b c , , 0;1a b c .

2 2 2 21 1 0,f u b c u b c b c với 20 1u a , , 0;1b c .

Page 39: Sinh viên Nguyễn Thị Lan Hƣơng · PDF fileLỜI CẢM ƠN Lời đầu tiên em xin bày tỏ lòng kính trọng và biết ơn sâu sắc đến Thầy giáo Thạc sĩ Nguyễn

KHÓA LUẬN TỐT NGHIỆP ĐẠI HỌC Nguyễn Thị Lan Hương – K36B SP Toán

35

0 0, , 0;1

1 0

fb c

f

.

+ Ta có 2 2 2 20 1 1 1 0f b c b c c c b , 0;1b c

2 21 0f b c b b , 0;1b c

Vậy 0, 0;1f u u (đpcm).

Ví dụ 8: Cho , , 0x y z và 1x y z . Chứng minh 2 2 2 4

27x y y z z x .

Hướng dẫn

+ Không mất tính tổng quát ta giả sử 1

min ; ; 03

x x y z x .

Khi đó, 2 2 2 2 2 2 21 1

3 3x y y z z x xy y z z x y z x y z

+ Yêu cầu bài toán

2 21 4 1, 0; , , 0, 1

3 27 3f x y z x y z x y z x y z

4

0 , , 0, 127

1 4 2, , 0,

3 27 3

f y z y z

f y z y z

.

+ Ta có

3

32 1 1 2 2 4 40 . .2

2 2 3 27 27

y zf y z y y z y z

, , 0, 1y z y z .

2 21 1 1

3 3 3f y z y z

.

Vì 1

3x thì

2 2

3 3y z z y nên

3 2 21 1 4 1 4 4

3 3 27 3 27 27f y y y y y y

Vậy ta có điều phải chứng minh.

Các bài tập luyện tập

1) Cho , ,a b c là ba cạnh của tam giác có chu vi bằng 1. Chứng minh

2 2 2 2 2a b c abc .

2) Cho , , 0x y z và 1x y z . Chứng minh 7 2 9xy yz xz xyz .

3) Cho , , 0x y z và 1x y z . Chứng minh 2 2 2 3 3 35 6 1x y z x y z .

4) Cho 1 , , 3x y z thỏa mãn 6x y z . Chứng minh rằng 2 2 2 14x y z .

Page 40: Sinh viên Nguyễn Thị Lan Hƣơng · PDF fileLỜI CẢM ƠN Lời đầu tiên em xin bày tỏ lòng kính trọng và biết ơn sâu sắc đến Thầy giáo Thạc sĩ Nguyễn

KHÓA LUẬN TỐT NGHIỆP ĐẠI HỌC Nguyễn Thị Lan Hương – K36B SP Toán

36

5) Cho 0 , , 2x y z thỏa mãn 3x y z . Chứng minh rằng 3 3 3 9x y z .

b. Một số ví dụ minh họa sử dụng phƣơng pháp tiếp tuyến

Ví dụ 1: Cho , , , 0a b c d thỏa mãn 4a b c d . Chứng minh

2 2 2 2

1

5 3 5 3 5 3 5 3 2

a b c d

a b c d

.

Hướng dẫn

+ Từ giả thiết suy ra , , , 0;4a b c d . Dấu đẳng thức xảy ra khi 1a b c d

+ Xét hàm số 25 3

tf t

t

+ Hãy viết phƣơng trình tiếp tuyến của đồ thị hàm số 25 3

tf t

t

tại điểm

11;

8M

?

Phƣơng trình tiếp tuyến của đồ thị hàm số là 1

332

y t .

+ Ta có

2

2 2

5 113 0, 0;4

5 3 32 32 5 3

t ttt t

t t

(*)

+ Thay , , ,a b c d vào t trong bất đẳng thức (*), cộng vế theo vế ta có đpcm.

Nhận xét

- Khi xét hiệu 0f t a t t b , ta thƣờng tách đƣợc nghiệm kép 0t t

(điểm dấu đẳng thức xảy ra).

- Khi trình bày lời giải, có thể ta không cần viết ra các giai đoạn tìm tiếp

tuyến mà đƣa ra luôn bất đẳng thức đặc trƣng cho bài toán cần chứng

minh.

Tƣơng tự, ta yêu cầu học sinh trình bày Ví dụ 2 và Ví dụ 3.

Ví dụ 2: Cho , , , 0a b c d thỏa mãn 4a b c d . Chứng minh rằng

3 3 3 34

2 2 2 2 27

a b c d

a b c d

.

Hướng dẫn

+ Từ giả thiết suy ra , , , 0;4a b c d . Dấu đẳng thức xảy ra khi nào?

Dấu bằng của đẳng thức xảy ra khi 1a b c d .

+ Hãy viết phƣơng trình tiếp tuyến của đồ thị hàm số 3

2

tf t

t

tại điểm

11;

27M

?

Page 41: Sinh viên Nguyễn Thị Lan Hƣơng · PDF fileLỜI CẢM ƠN Lời đầu tiên em xin bày tỏ lòng kính trọng và biết ơn sâu sắc đến Thầy giáo Thạc sĩ Nguyễn

KHÓA LUẬN TỐT NGHIỆP ĐẠI HỌC Nguyễn Thị Lan Hương – K36B SP Toán

37

Phƣơng trình tiếp tuyến của đồ thị hàm số là 2 1

27 27y t .

+ Ta chứng minh 3

2 1, 0;4

2 27

t tt

t

(*)

2 22 1 6 4 0t t t 0;4t

(luôn đúng vì 2 6 4 4 2 4 0, 0;4t t t t t t )

+ Thay , , ,a b c d vào t trong bất đẳng thức (*), cộng vế theo vế ta có đpcm.

Ví dụ 3: Cho , , 0a b c thỏa mãn 1a b c . Chứng minh rằng

2 2 2

9

1 1 1 10

a b c

a b c

.

Hướng dẫn

+ Dấu đẳng thức xảy ra khi nào?

Dấu đẳng thức xảy ra khi 3

1 cba

+ Xét hàm 21

tf t

t

trên 0;1

+ Viết phƣơng trình tiếp tuyến của đồ thị hàm số f t tại điểm 1 3

;3 10

M

?

Phƣơng trình tiếp tuyến là 18 3

25 50y t .

+ Ta chứng minh 2

18 3, 0;1

1 25 50

tt t

t

.

Ví dụ 4: Cho , , , 0a b c d thỏa mãn 1a b c d . Chứng minh

3 3 3 3 2 2 2 2 16

8a b c d a b c d .

Hướng dẫn

+ Bất đẳng thức cần chứng minh có dạng

3 2 3 2 3 2 3 2 16 6 6 6

8a a b b c c d d (*)

+ Vai trò của cba ,, là nhƣ nhau. Dấu của đẳng thức xảy ra khi nào?

Dấu đẳng thức xảy ra khi 4

1 dcba

+ Viết phƣơng trình tiêpd tuyến của đồ thị hàm số 3 26f t t t tại điểm

1 1;

4 32M

?

Phƣơng trình tiếp tuyến là 5 1

8

ty

.

Page 42: Sinh viên Nguyễn Thị Lan Hƣơng · PDF fileLỜI CẢM ƠN Lời đầu tiên em xin bày tỏ lòng kính trọng và biết ơn sâu sắc đến Thầy giáo Thạc sĩ Nguyễn

KHÓA LUẬN TỐT NGHIỆP ĐẠI HỌC Nguyễn Thị Lan Hương – K36B SP Toán

38

+ Ta có

23 2 5 1 1

6 4 1 3 1 0, 0;18 8

tt t t t t

5 1, 0;1

8

tf t t

(*).

+ Thay , , ,a b c d vào t trong bất đẳng thức (*), cộng vế theo vế ta có đpcm.

Áp dụng Ví dụ 4, yêu cầu học sinh tƣơng tự làm Ví dụ 5.

Ví dụ 5: Cho , , 0a b c thỏa mãn 3a b c . Chứng minh

2 2 2

2 2 22 2 2

9 9 95

2 2 2

a b c

a b c b c a c a b

.

Hướng dẫn

+ Bất đẳng thức cần chứng minh tƣơng đƣơng với

2 2 2

2 2 22 2 2

9 9 95

2 3 2 3 2 3

a b c

a a b b c c

.

+ Hãy viết phƣơng trình tiếp tuyến của đồ thị hàm số

2

22

9

2 3

tf t

t t

tại điểm

51;

3M

?

Phƣơng trình tiếp tuyến là 4

3

ty

.

+ Hãy chứng minh

2

22

9 4, 0;3

32 3

t tt

t t

Thật vậy

2 2

2 222

9 9 1 2 6 1 2 6 4

3 6 9 3 3 6 32 3 3 1 6

t t t t t

t tt t t

.

Nhận xét

Qua các ví dụ trên ta thấy việc xác định dấu của biểu thức

0f t a t t b trên D có thể làm nhƣ sau:

- Dựa vào dấu của bất đẳng thức cần chứng minh.

- Dự đoán bằng cách thay một giá trị bất kì của t D vào biểu thức

0f t a t t b .

- Phân tích 2

0 0 .f t a t t b t t h t và xác định dấu của h t trên

D .

Trong các ví dụ trên ta đều sử dụng cách 3. Tuy nhiên trong một số bài toán

việc phân tích nhƣ trên gặp khó khăn vì bài toán chứa căn thức. Do đó, ta có thể

gợi ý cho học sinh sử dụng phƣơng pháp hàm số, tận dụng luôn kết quả mà các

Page 43: Sinh viên Nguyễn Thị Lan Hƣơng · PDF fileLỜI CẢM ƠN Lời đầu tiên em xin bày tỏ lòng kính trọng và biết ơn sâu sắc đến Thầy giáo Thạc sĩ Nguyễn

KHÓA LUẬN TỐT NGHIỆP ĐẠI HỌC Nguyễn Thị Lan Hương – K36B SP Toán

39

em tính đạo hàm của hàm f t khi lập phƣơng trình tiếp tuyến và chú ý đạo hàm

của hàm số 0f t a t t b vẫn có nghiệm 0t t . Ta xét tiếp ví dụ sau:

Ví dụ 6: Cho số thực dƣơng , ,x y z thỏa mãn 1x y z . Chứng minh rằng

6

21 1 1

x y z

x y z

Hướng dẫn:

+ Phƣơng trình tiếp tuyến của đồ thị hàm số 1

xf x

x

tại điểm

1 1;

3 6M

3

15 112 2

y x .

+ Ta đi chứng minh 3

15 1 , 0;11 12 2

xx x

x

Xét hàm số 3

15 11 12 2

xg x x

x

trên 0;1

1

' 03

g x x . Từ bảng biến thiên ta có điều phải chứng minh.

Các ví dụ trên đều cần có giả thiết 1 2 ... na a a n , để sử dụng phƣơng

pháp tiếp tuyến. Tuy nhiên trong các bài toán, có sự đồng bậc của tử và mẫu

trong từng số hạng hoặc đồng bậc của hai vế bất đẳng thức cần chứng minh, ta

vẫn có thể nghĩ đến phƣơng pháp tiếp tuyến nhờ việc chuẩn hóa bài toán. Ta xét

ví dụ sau:

Ví dụ 7: Cho , , 0a b c . Chứng minh rằng

2 2 2

2 2 22 2 2

2 2 28

2 2 2

a b c b c a c a b

a b c b c a c a b

.

Hướng dẫn

+ Do mỗi số hạng có tử và mẫu là các biểu thức đẳng cấp nên không mất tính

tổng quát ta giả sử 3a b c . Khi đó, bất đẳng thức cần chứng minh là 2 2 2

2 2 2

6 9 6 9 6 924

2 3 2 3 2 3

a a b b c c

a a b b c c

.

+ Phƣơng trình tiếp tuyến của đồ thị hàm số 2

2

6 9

2 3

t tf t

t t

tại điểm 1;8M là

4 4y t .

+ Ta đi chứng minh 2

2

6 94 4, 0;3

2 3

t tt t

t t

.

Ví dụ 8: Cho , , 0a b c . Chứng minh rằng

Page 44: Sinh viên Nguyễn Thị Lan Hƣơng · PDF fileLỜI CẢM ƠN Lời đầu tiên em xin bày tỏ lòng kính trọng và biết ơn sâu sắc đến Thầy giáo Thạc sĩ Nguyễn

KHÓA LUẬN TỐT NGHIỆP ĐẠI HỌC Nguyễn Thị Lan Hương – K36B SP Toán

40

2 2 2

9

4

a b ca b c

b c c a a b

.

Hướng dẫn

+ Do mỗi số hạng có tử và mẫu là các biểu thức đẳng cấp nên không mất tính

tổng quát ta giả sử 3a b c . Khi đó, bất đẳng thức cần chứng minh có dạng

2 2 2

3

43 3 3

a b c

a b c

+ Phƣơng trình tiếp tuyến của đồ thị hàm số

23

tf t

t

tại điểm 1

1;4

M

?

Phƣơng trình tiếp tuyến là 2 1

4

ty

.

+ Ta chứng minh

2

2 1, 0;3

43

t tt

t

2

2

1 9 20, 0;3

4 3

t tt

t

(luôn đúng).

Ví dụ 9: Cho , , 0a b c thỏa mãn 1abc . Chứng minh

2 2 21 1 1

2 2 2

a b ca b c

.

Hướng dẫn

Ta có 1 ln ln ln 0abc a b c . Đặt ln , ln , lnx a y b z c .

Khi đó ta có bài toán

Cho các số thực , ,x y z thỏa mãn 0x y z . Chứng minh

2 2 21 1 10

2 2 2

x y zx y ze e e

e e e

.

+ Xét hàm số 2 1

2

tte

f t e

trên R

+ Phƣơng trình tiếp tuyến của đồ thị hàm số f t tại điểm 0;0O ?

Phƣơng trình tiếp tuyến là 1

2y t

+ Ta đi chứng minh Rttee t

t

,2

1

2

12

.

Xét hàm

2 2

2

1 1 1' 0 0

2 2 22 1

t tt t

t

e eg t e t g t e t

e

.

Trong một số bài toán việc xét hiệu 0f x a x x b mặc dù tách

đƣợc nghiệm bội 0x x , nhƣng hiệu đó không giữ nguyên một dấu trên D .

Page 45: Sinh viên Nguyễn Thị Lan Hƣơng · PDF fileLỜI CẢM ƠN Lời đầu tiên em xin bày tỏ lòng kính trọng và biết ơn sâu sắc đến Thầy giáo Thạc sĩ Nguyễn

KHÓA LUẬN TỐT NGHIỆP ĐẠI HỌC Nguyễn Thị Lan Hương – K36B SP Toán

41

Trong trƣờng hợp đó, ta có thể chia trƣờng hợp để chứng minh. Ta xét một số ví

dụ sau:

Ví dụ 10: Cho , , 0a b c thỏa mãn 3a b c . Chứng minh

2 2 2

2 2 2

1 1 1a b c

a b c .

Hướng dẫn

Nhận xét: Nếu xét hàm số 2

2

1f t t

t trên 0;3 thì phƣơng trình tiếp

tuyến tại điểm 1;0M là 4 4y t

Khi đó 2

2

2 2

1 2 1 1 214 4

t t tt t

t t

lúc âm, lúc dƣơng trên

0;3 . Do đó ta cần chia khoảng giá trị cho các biến , ,a b c .

Hướng dẫn:

Cách 1.

+ Nếu 3 2,4a thì 0,6b c . Khi đó, 22 2 2 9VP a b c a b c .

2 22 2 2 2

1 1 1 1 2 1 2 1 29

3 9 9 0,3

2

VTa b c bc b c

.

+ Ta chỉ xét các số , , 0;2,4a b c . Khi đó, sử dụng phƣơng trình tiếp tuyến ta có

2

2

2 2

1 2 1 1 214 4 0, 0;2,4

t t tt t t

t t

.

Cách 2.

Ta thấy nếu có một số trong ba số , ,a b c nhỏ hơn 1

3 khi đó

2 2 2 2

2 2 2

1 1 19 a b c a b c

a b c , bất đẳng thức luôn đúng.

Vậy ta chỉ xét 1

, ,3

a b c . Vì 1 7

3 , , ;3 3

a b c a b c

.

Khi đó

2 2

2

2 2

1 2 11 1 74 4 0, ;

3 3

t tt t t

t t

.

Các bài tập luyện tập

1. Cho , ,a b c là độ dài ba cạnh một tam giác. Chứng minh rằng:

1 1 1 9 1 1 14

a b c a b c a b b c c a

.

Page 46: Sinh viên Nguyễn Thị Lan Hƣơng · PDF fileLỜI CẢM ƠN Lời đầu tiên em xin bày tỏ lòng kính trọng và biết ơn sâu sắc đến Thầy giáo Thạc sĩ Nguyễn

KHÓA LUẬN TỐT NGHIỆP ĐẠI HỌC Nguyễn Thị Lan Hương – K36B SP Toán

42

2. Cho các số thực , ,x y z thỏa mãn 0x y z . Chứng minh

3 2

21 1 1

x y z

x y z

e e e

e e e

3. Cho , , 0a b c thỏa mãn 1abc . Chứng minh 2 2 2

1 1 11

1 1 1a a b b c c

.

4. Cho , , 0a b c . Chứng minh

2 2 2

2 2 22 2 2

3

5

a b c

a b c b a c c b a

5. Cho , , 0a b c thỏa mãn 1abc . Chứng minh 3 2

21 1 1

a b c

a b c

.

c. Một số ví dụ minh họa sử dụng đạo hàm

Ví dụ 1: Cho ba, là hai số thực không âm thỏa: 4 ba . Chứng minh rằng

5

11

2

4

1

1

7

9

ba.

Hướng dẫn

+ Từ giả thiết có nhận xét gì về mối liên hệ giữa các đại lƣợng ba, ?

Bài toán có sự liên hệ giữa hai đại lƣợng ba, thông qua 2 điều kiện cơ

bản: ba, không âm và 4 ba .

+ Ta có thể biểu diễn một trong hai đại lƣợng thông qua đại lƣợng còn lại không?

Ta hoàn toàn có thể biểu diễn đƣợc a thông qua b hoặc b thông qua a .

Ta có abba 44 . Từ các ràng buộc của giả thiết ta có: 4;0a .

Khi đó ta có: aaba

6

4

1

1

2

4

1

1

Đặt aa

af

6

4

1

1.

+ Hãy khảo sát hàm số aa

af

6

4

1

1với 4;0a ?

Ta có:

22

6

4

1

1'

aaaf

. Cho

3

4

8

06

4

1

10'

22 a

a

aaaf

Từ đó ta có: 5

11

7

9 af

Vậy 5

11

2

4

1

1

7

9

ba.

Và 2

4

1

1

7

9

ba khi

3

8

3

4

b

a

, 5

11

2

4

1

1

ba khi

0

4

b

a

Page 47: Sinh viên Nguyễn Thị Lan Hƣơng · PDF fileLỜI CẢM ƠN Lời đầu tiên em xin bày tỏ lòng kính trọng và biết ơn sâu sắc đến Thầy giáo Thạc sĩ Nguyễn

KHÓA LUẬN TỐT NGHIỆP ĐẠI HỌC Nguyễn Thị Lan Hương – K36B SP Toán

43

Ví dụ 2: Cho 2;3; ba đồng thời thỏa: 233 ba . Chứng minh rằng:

32 3642

ba

Hướng dẫn

+ Dựa vào giả thiết đề bài cho, hãy biểu diễn a thông qua b hoặc b thông qua a?

Ta có: 3 333 22 abba . Với các ràng buộc của giả thiết ta có:

2;63a

Khi đó có VT của bất đẳng thức cần chứng minh trở thành

3 32 2 aaP .

+ Để bài toán đơn giản ta đặt: 3ax , khi đó điều kiện của x là gì?

Điều kiện x [ 6;8] .

+ Hãy xét hàm số: 3 2 23f (x) x (2 x) với x [ 6;8] ?

Có 3 3

2 2f '(x)

3 t 3 2 t

; f '(x) 0 x 1

Từ BBT của f(x) ta có: 3 34 f (x) 4 36

Vậy: GTNN của hàm số là 3 4 đạt đƣợc khi 3

3

x 0 x 2;

y 2 y 0

; GTLN của

hàm số là 34 36 đạt đƣợc khi 3

3

x 2 x 3;

y 3 y 2

+ Từ đây ta có kết luận gì?

Kết luận: Bất đẳng thức đƣợc chứng minh.

Dấu đẳng thức xảy ra khi

3 3

2

y

x;

2

33

y

x

Ví dụ 3: (ĐH Khối A-2011). Cho ba số thực , , 1;4x y z và ,x y x z . Tìm giá

trị nhỏ nhất của biểu thức : 2 3

x y zP

x y y z z x

Hướng dẫn

Trƣớc khi đi vào trình bày lời giải ta cùng nhau xem đáp án của bộ :

Chứng minh bất đẳng thức :

abba 1

2

1

1

1

1 với 0, ba và

1. ba

Điều này dễ dàng thực hiện đƣợc nhờ phép biến đổi tƣơng đƣơng.

Áp dụng (*) cho bài toán ta có :

Page 48: Sinh viên Nguyễn Thị Lan Hƣơng · PDF fileLỜI CẢM ƠN Lời đầu tiên em xin bày tỏ lòng kính trọng và biết ơn sâu sắc đến Thầy giáo Thạc sĩ Nguyễn

KHÓA LUẬN TỐT NGHIỆP ĐẠI HỌC Nguyễn Thị Lan Hương – K36B SP Toán

44

x 1 1 1 2P

z x 3y2x 3y x1 1 2 1y z x y

Đặt x

t ; t [1;2]y

khi đó có : 2

2

t 2P

1 t2t 3

Xét hàm 2

2

t 2f (t)

1 t2t 3

với t [1;2] ta có 34

f (t) f (2)33

Vậy 33

34min P khi 4, 1, 2 x y z

* Nhận xét: Bài toán trên căn bản là tƣơng đối khó đối với mức độ của

một đề thi đại học. Ngoài việc phải biến đổi P để nhận ra dạng của bất đẳng

thức phụ thì việc giải bài toán đạo hàm ở phần sau cũng rất phức tạp.

Chúng ta cùng xem xét một cách giải khác có vẻ tự nhiên và thích hợp

hơn.

Ta có : 2 3

x y zP

x y y z z x

.

+ Hãy khảo sát hàm số trên nhƣ là hàm theo biến z ; còn ,x y là hằng số ?

Ta có: 2

2 2 2 2

( )( )'( )

( ) ( ) ( ) ( )

y z x y z xyP z

y z z x y z z x

Theo giả thiết 0x y x y nên xyzP 0' (do , , 1;4x y z )

Lập BBT của zP ta đƣợc: 2 2

( ) =2 3

2 3 1

x

yx yP P xy

xx y x y x

y y

Đặt xt

y

+ Ta có hàm số nào?

Ta có hàm số 2

2

2( )

2 3 1

tf t

t t

+ Hãy tìm điều kiện của ẩn?

Do ,x y x z và , , 1;4x y z nên 1 2t .

+ Khảo sát hàm số 2

2

2( )

2 3 1

tf t

t t

với 1 2t ?

Xét hàm 2

2

2( )

2 3 1

tf t

t t

.

Page 49: Sinh viên Nguyễn Thị Lan Hƣơng · PDF fileLỜI CẢM ƠN Lời đầu tiên em xin bày tỏ lòng kính trọng và biết ơn sâu sắc đến Thầy giáo Thạc sĩ Nguyễn

KHÓA LUẬN TỐT NGHIỆP ĐẠI HỌC Nguyễn Thị Lan Hương – K36B SP Toán

45

Ta có 3 2

2 2 2

2 4 ( 1) 3(2 3)'( ) 0, 1;2

(2 3) (1 )

t t t tf t t

t t

.Suy ra ( )f t giảm trên

1;2 , do đó 34

( ) ( ) (2)33

P P xy f t f

Đẳng thức xảy ra : 4, 1, 22

z xy

x y zxt

y

.

Vậy 34

min33

P khi 4, 1, 2 x y z

Ví dụ 4: ( Đề thi HSG Ninh Bình 2010)

Cho cba ,, là độ dài ba cạnh của tam giác có chu vi bằng 3. Tìm giá trị

nhỏ nhất của biểu thức: 2 2 23( ) 4T a b c abc

Hướng dẫn

Do vai trò bình đẳng của cba ,, nên ta có thể giả sử : 0 a b c .

Vì chu vi bằng 3 nên a + b + c=3 nên 3a b c mà

31

2a b c c

+ Coi T là hàm theo biến c .

Ta biến đổi : 2 2 2 2 2 23( ) 4 3( ) 3 4 T a b c abc a b c abc

2 2 2 2=3 ( ) 2 3 4 3(3 ) 3 2 (2 3) a b ab c abc c c ab c

Mặt khác : 2 2

3

2 2

a b cab

23

(2 3) (2 3)2

cab c c

( vì 3

2 3 02

c c )

Do đó : 2

2 2 3 23 3 273(3 ) 3 2(2 3) ( )

2 2 2

cT c c c c c f c

+ Hãy xét hàm số 3 23 27( )

2 2f c c c , trên

31;

2

?

2'( ) 3 3 ; '( ) 0 1 f c c c f c c

Khi đó từ bảng biến thiên suy ra: ( ) (1) 13f c f .

Suy ra ( ) (1) 13T f c f khi 1 cba

Vậy 113min cbaP .

Page 50: Sinh viên Nguyễn Thị Lan Hƣơng · PDF fileLỜI CẢM ƠN Lời đầu tiên em xin bày tỏ lòng kính trọng và biết ơn sâu sắc đến Thầy giáo Thạc sĩ Nguyễn

KHÓA LUẬN TỐT NGHIỆP ĐẠI HỌC Nguyễn Thị Lan Hương – K36B SP Toán

46

* Ngoài cách giải bằng phƣơng pháp đạo hàm theo một biến đã đƣợc trình

bày ở trên ta còn có rất nhiều cách để giải bài toán này. Tuy nhiên theo quan

điểm của tôi thì cách giải sau có lẽ là hợp lý hơn cả.

+ Từ các giả thiết ta có điều kiện của các số tham gia trong bất đảng thức là gì?

2

3,,0 cba

+ Biến đổi: bcacbaabccabcabcbaT 646274632

Hay: bcaaaT 6427186 2

Do 0642

3 aa . Sử dụng đánh giá:

2

3

2

22acb

bc

ta đƣa về biểu

thức một biến dạng:

afa

aaaP

2

36427186

2

2

Khảo sát hàm af với 2

30 a ta có 131 faf

Vậy 13min P khi 1 cba .

Ví dụ 5: Cho , ,a b c là ba số thực thỏa mãn điều kiện abc a c b . Tìm giá trị

lớn nhất của biểu thức : 2 2 2

2 2 3

1 1 1P

a b c

Hướng dẫn

+ Ta thử giảm dần biến số và thực hiện phép toán lấy đạo hàm ?

Theo giả thiết ta có (1 ) 01

a ca c b ac b

ac

1a

c .

Thay vào biểu thức P ta đƣợc : 2

2 2 2 2

2 2( ) 3 12 , (0 )

1 ( 1)( 1) 1

a cP a

a a c c c

+ Xét hàm số:2

2 2 2

1 ( )( ) 1

1 ( 1)( 1)

x cf x

x x c

với

10 x

c và coi c là tham số

c >0 ?

Ta có : 2

2

02 2 2

2 ( 2 1) 1'( ) 0 1 0;

(1 ) (1 )

c x cxf x x c c

x c c

Từ bảng biến thiên ta có : 02

( ) ( )1

cf x f x

c

.

2 22

3 2 32 ( ) ( )

1 11

cS f a g c

c cc

+ Hãy xét hàm số )(cg ?

Ta có : 2

02 2 2

2(1 8 ) 1'( ) 0 0;

8(1 ) (3 1 )

cg c c c

c c c

Page 51: Sinh viên Nguyễn Thị Lan Hƣơng · PDF fileLỜI CẢM ƠN Lời đầu tiên em xin bày tỏ lòng kính trọng và biết ơn sâu sắc đến Thầy giáo Thạc sĩ Nguyễn

KHÓA LUẬN TỐT NGHIỆP ĐẠI HỌC Nguyễn Thị Lan Hương – K36B SP Toán

47

Từ bảng biến thiên suy ra : 0( ) ( )g c g c 0

10( ) ( )

3S g c g c .

+ Kết luận: Vậy với 1 2

, , 228

c a b thì 10

ax3

M S .

Các bài tập luyện tập

1. Cho ba, là các số thực dƣơng thỏa: 4

5 ba . Chứng minh rằng: 5

4

14

ba

2. Cho x,y,z [ 1;1] và xy yz zx 0 . Tìm GTNN của biểu thức:

2 2P y z 4x 2y 2z

3. Cho ba số dƣơng , ,a b c thỏa mãn điều kiện 21 2 8 12ab bc ca . Tìm giá trị

nhỏ nhất của biểu thức: 1 2 3

Pa b c

.

4. Cho zyx ,, là các số thực không âm thỏa: 2 2 2x y z 1 . Chứng minh rằng:

27

292

zxyxyzP

5. Cho các số dƣơng ba, thỏa mãn: 2ba . Tìm GTNN của biểu thức:

269

1

269

12424

bbaa

P

2.2.3. Phƣơng pháp chứng minh bất đẳng thức dựa vào cách đặt ẩn phụ

Trong nhiều bài toán, ta đặt ẩn phụ để đƣa bài toán về bài toán quen thuộc

hơn.

Ví dụ 1: Chứng minh rằng nếu 6,5,4 cba và 90222 cba thì

16 cba .

Hướng dẫn

+ Mối liên hệ giữa các đại lƣợng a, b, c đƣợc xác định nhƣ thế nào?

6,5,4 cba và 90222 cba

+ Với mối liên hệ này thì có thể biểu diễn các đại lƣợng qua biến số trung gian

nhƣ thế nào?

Đặt zcybxa 6,5,4 , ta có 0,, zyx

+ Hãy chuyển bất đẳng thức cần chứng minh theo ẩn mới?

1 zyx

+ Chứng minh bất đẳng thức này nhƣ thế nào?

Nếu trong ba số zyx ,, có ít nhất một số lớn hơn 1 thì zyx 1 vì

0,, zyx .

Nếu 1,, zyx thì

Page 52: Sinh viên Nguyễn Thị Lan Hƣơng · PDF fileLỜI CẢM ƠN Lời đầu tiên em xin bày tỏ lòng kính trọng và biết ơn sâu sắc đến Thầy giáo Thạc sĩ Nguyễn

KHÓA LUẬN TỐT NGHIỆP ĐẠI HỌC Nguyễn Thị Lan Hương – K36B SP Toán

48

zyx

zyxzyxzyxzyxzyx

1

131212121210813 222

Nhƣ vậy trong mọi trƣờng hợp ta có 1 zyx

+ Đẳng thức xảy ra khi nào?

Đẳng thức xảy ra khi 0 yx và 1z , tức là 7,5,4 cba .

Ví dụ 2: Cho 2 yx . Chứng minh rằng 255 yx .

Hướng dẫn

+ Mối liên hệ giữa các đại lƣợng đƣợc xác định nhƣ thế nào?

2 yx

+ Với mối liên hệ này thì có thể biểu diễn các đại lƣợng qua một biến số trung

gian nhƣ thê nào?

Đặt Raayax ,1,1

+ Hãy chuyển bất dẳng thức cần chứng minh theo ẩn mới?

21155 aa

+ Chứng minh bất đẳng thức này nhƣ thế nào?

Bất đẳng thức cần chứng minh tƣơng đƣơng với 210202 42 aa (bất

đẳng thức này đúng với mọi a )

+ Đẳng thức xảy ra khi nào?

Đẳng thức xảy ra khi 0a hay 1 yx

Ví dụ 3: Các số dƣơng zyx ,, thỏa mãn 4222 xyzzyx .

Chứng minh rằng 3 zyx .

Hướng dẫn

+ Đẳng thức 4222 xyzzyx 1222

2222

222

zyxzyx, nhƣ

vậy đẳng thức tƣơng đƣơng với hệ thức lƣợng giác nào giữa các góc trong một

tam giác?

1coscoscos2coscoscos 222 CBsACBA

+ Có thể biểu diễn các đại lƣợng zyx ,, qua các hàm số lƣợng giác tƣơng ứng

với đẳng thức trên đƣợc không?

Chọn

2;0,,

CBA sao cho Cz

By

Ax

cos2

,cos2

,cos2

Theo giả thiết

1coscoscos2coscoscos 222 CBsACBA CBA

+ Bất đẳng thức cần chứng minh đƣợc chuyển về bất đẳng thức lƣợng giác nào?

3cos2cos2cos2 CBA (*)

+ Hãy chứng minh bất đẳng thức (*) này?

Page 53: Sinh viên Nguyễn Thị Lan Hƣơng · PDF fileLỜI CẢM ƠN Lời đầu tiên em xin bày tỏ lòng kính trọng và biết ơn sâu sắc đến Thầy giáo Thạc sĩ Nguyễn

KHÓA LUẬN TỐT NGHIỆP ĐẠI HỌC Nguyễn Thị Lan Hương – K36B SP Toán

49

Ta có:

2

3

2

11

2cos

2

1

2cos

2

1

2sin21

2sin21

2cos

2sin2cos

2cos

2cos2coscoscos

2

2

2

BABAC

CBACC

BABACBA

Vậy ta có bất đẳng thức (*) đƣợc chứng minh. Tức là bất đẳng thức đã cho

ban đầu đã đƣợc chứng minh.

Đẳng thức xảy ra khi 3

CBA hay 1 zyx

Ví dụ 4: Cho zyx ,, dƣơng thỏa mãn xyzzyx . Chứng minh rằng

2

3

1

1

1

1

1

1

222

zyx

Hướng dẫn

+ Đẳng thức xyzzyx tƣơng ứng với hệ thức lƣợng giác nào giữa các góc

trong một tam giác?

CBACBA tantantantantantan

+ Có thể biểu diễn các đại lƣợng zyx ,, qua các hàm số tƣơng ứng với đẳng thức

trên đƣợc không?

Cho

2;0,,

CBA sao cho CzByAx tan,tan,tan

Theo giả thiết CBACBA tantantantantantan CBA

+ Bất đẳng thức cần chứng minh đƣợc chuyển về bất đẳng thức lƣợng giác nào?

2

3

tan1

1

tan1

1

tan1

1

222

CBA hay

2

3coscoscos CBA (**)

Bất đẳng thức (**) đã đƣợc chứng minh ở bài trên.

Đẳng thức xảy ra khi 3

CBA

Vậy 2

3

1

1

1

1

1

1

222

zyx

Đẳng thức xảy ra khi 3 zyx

Ví dụ 5: Cho các số dƣơng cba ,, thỏa mãn bcaabc . Chứng minh rằng:

3

10

1

3

1

2

1

2222

cba

.

Hướng dẫn

+ Đẳng thức bcaabc 11

.1

. b

cb

aac tƣơng ứng với hệ thức lƣợng giác

nào giữa các góc trong một tam giác?

Page 54: Sinh viên Nguyễn Thị Lan Hƣơng · PDF fileLỜI CẢM ƠN Lời đầu tiên em xin bày tỏ lòng kính trọng và biết ơn sâu sắc đến Thầy giáo Thạc sĩ Nguyễn

KHÓA LUẬN TỐT NGHIỆP ĐẠI HỌC Nguyễn Thị Lan Hương – K36B SP Toán

50

12

tan2

tan2

tan2

tan2

tan2

tan ACCBBA

+ Có thể biểu diễn các đại lƣợng cba ,, qua các hàm số lƣợng giác tƣơng ứng với

đẳng thức trên đƣợc không?

Cho ;0,, CBA sao cho 2

tan,2

tan1

,2

tanC

cB

b

Aa

Theo giả thiết 12

tan2

tan2

tan2

tan2

tan2

tan ACCBBA

CBA

+ Bất đẳng thức cần chứng minh đƣợc chuyển về bất đẳng thức lƣợng giác nào?

6

11cos

2

3coscos

3

10

2cos3

2sin2

2cos2 222 CBA

CBA

+ Chứng minh bất đẳng thức lƣợng giác này nhƣ thế nào?

Bất đẳng thức cần chứng minh tƣơng đƣơng với

02

cos9

1

2cos

3

1

2sin

6

11

2sin21

2

3

2cos

2sin2 2

2

2

BABACCBAC

Bất đẳng thức này đúng, suy ra bất đẳng thức cần chứng minh.

Đẳng thức xảy ra khi BA và 3

1

2sin

C hay

8

1,2,

2

2 cba

Rút ra kinh nghiệm: Nếu giả thiết của bài toán tƣơng ứng với một hệ

thức lƣợng giác nào đó thì có thể đặt ẩn phụ đƣa về các bất đẳng thức tam giác.

Ví dụ 6: Cho zyx ,, là các số thực khác 1 thỏa mãn 1xyz . Chứng minh rằng

1

1112

2

2

2

2

2

z

z

y

y

x

x.

Hướng dẫn

+ Nếu coi các đại lƣợng 1

,1

,1 z

z

y

y

x

x là các biến số trung gian a,b,c. Hãy tìm hệ

thức liên hệ giữa cba ,, ?

Đặt 1

11,

1

11,

1

11

1,

1,

1

zc

yb

xa

z

zc

y

yb

x

xa

Ta có:

abczyx

xyz

zyxcba

111111

1111 (vì 1xyz )

101 cbacabcabcbacabcab

+ Hãy chuyển bất đẳng thức cần chứng minh theo biến trung gian?

1222 cba

+ Chứng minh bất đẳng thức trung gian này nhƣ thế nào?

Ta có:

111

222

2

22222

cba

cbacbacabcabcbacba

Page 55: Sinh viên Nguyễn Thị Lan Hƣơng · PDF fileLỜI CẢM ƠN Lời đầu tiên em xin bày tỏ lòng kính trọng và biết ơn sâu sắc đến Thầy giáo Thạc sĩ Nguyễn

KHÓA LUẬN TỐT NGHIỆP ĐẠI HỌC Nguyễn Thị Lan Hương – K36B SP Toán

51

Bất đẳng thức trung gian đƣợc chứng minh.

Vậy ta có bất đẳng thức cần chứng minh.

2.2.4. Phƣơng pháp chứng minh bất đẳng thức bằng cách kết hợp nhiều

phƣơng pháp khác nhau

Ví dụ 1 : Cho 51222 ba . Chứng minh 102 ba .

Hướng dẫn

+ Mối liên hệ giữa các đại lƣợng 2a và 1b đƣợc xác định nhƣ thế nào ?

51222 ba .

+ Với mối liên hệ này thì có thể biểu diễn các đại lƣợng qua các hàm số lƣợng

giác nhƣ thế nào ?

Đặt sin51;cos.52 ba .

+ Bất đẳng thức cần chứng minh đƣợc chuyển về bất đẳng thức lƣợng giác nào?

Trƣớc hết, ta có: sin51;cos.52 ba .

Ta phải chứng minh 52 ba 10)sin51()cos52(2 .

+ Chứng minh bất đẳng thức lƣợng giác này nhƣ thế nào?

Áp dụng Bất đẳng thức 22cos.sin. BABA .

105525)sin5cos52(522

đpcm.

Nhận xét

Áp dụng Bất đẳng thức 22cos.sin. BABA ta có kết quả

02 ba .

Ví dụ 2: Chứng minh bất đẳng thức 98161 22 aaa , 1;1a .

Hướng dẫn

+ Với điều kiện a 1;1 thì đại lƣợng này đƣợc biểu diễn qua hàm số lƣợng

giác nhƣ thế nào?

Đặt cosx , ;0 .

+ Bất đẳng thức cần chứng minh đƣợc chuyển về bất đẳng thức lƣợng giác nào?

9cos8cos1.cos61 22 .

+ Chứng minh bất đẳng thức lƣợng giác này nhƣ thế nào?

sin1 2 a .

42cos42sin3)2cos1(4sin.cos6cos8cos1.cos6 22 .

Áp dụng bất đẳng thức 2222 cos.sin. baxbxaba ta có

942cos42sin3144342cos42sin3443 2222

Page 56: Sinh viên Nguyễn Thị Lan Hƣơng · PDF fileLỜI CẢM ƠN Lời đầu tiên em xin bày tỏ lòng kính trọng và biết ơn sâu sắc đến Thầy giáo Thạc sĩ Nguyễn

KHÓA LUẬN TỐT NGHIỆP ĐẠI HỌC Nguyễn Thị Lan Hương – K36B SP Toán

52

Ví dụ 3: Cho 4 số dƣơng dcba ,,, thỏa mãn 1 1 1 11

1 1 1 1a b c d

.

Chứng minh abcd 81.

Hướng dẫn

+ Mối liên hệ giữa các đại lƣợngdcba 1

1;

1

1;

1

1;

1

1 đƣợc xác định nhƣ thế

nào ?

11

1

1

1

1

1

1

1

dcba.

+ Với mối liên hệ này thì có thể biểu diễn các đại lƣợng qua bốn biến số trung

gian nhƣ thế nào?

Tồn tại tzyx ,,, dƣơng thỏa mãn

1 1 1 1

; ; ;1 1 1 1

x y z t

a x y z t b x y z t c x y z t d x y z t

.

+ Bất đẳng thức cần chứng minh về bất đẳng thức nào với bốn biến số trung

gian ?

Ta có ; ; ;y z t z t x t x y x y z

a b c dx y z t

.

Bất đẳng thức cần chứng minh trở thành

81y z t z t x t x y x y z

xyzt

.

+ Chứng minh bất đẳng thức bốn biến số trung gian nhƣ thế nào?

Áp dụng bất đẳng thức Côsi cho ba số dƣơng

3333 3,3,3,3 xyzzyxtxyyxtztxxtzyzttzy .

Từ đó có đpcm.

Tổng quát: Cho 4 số dƣơng dcba ,,, và 0m thỏa mãn

1 1 1 1

11 1 1 1ma mb mc md

.

Chứng minh abcd4

81

m .

Tồn tại tzyx ,,, dƣơng thỏa mãn

1 1 1 1; ; ;

1 1 1 1

x y z t

ma x y z t mb x y z t mc x y z t md x y z t

Page 57: Sinh viên Nguyễn Thị Lan Hƣơng · PDF fileLỜI CẢM ƠN Lời đầu tiên em xin bày tỏ lòng kính trọng và biết ơn sâu sắc đến Thầy giáo Thạc sĩ Nguyễn

KHÓA LUẬN TỐT NGHIỆP ĐẠI HỌC Nguyễn Thị Lan Hương – K36B SP Toán

53

; ; ;y z t z t x t x y x y z

a b c dmx my mz mt

.

4 4

81y z t z t x t x y x y zabcd

m xyzt m

.

Ví dụ 4 : Indian Mathematical Olympiad 2003.

Cho hai số yx, không âm thỏa mãn 2 yx .

Chứng minh 23333 yxyx .

Hướng dẫn

+ Mối liên hệ giữa các đại lƣợng yx, đƣợc xác định nhƣ thế nào ?

2 yx .

+ Với mối liên hệ này thì có thể biểu diễn các đại lƣợng qua một biến số trung

gian nhƣ thế nào?

tx 1 , ty 1 11 t .

+ Bất đẳng thức cần chứng minh về bất đẳng thức nào với biến số trung gian ?

211.1.13333 tttt .

262.1 232 tt .

262.22 232 tt .

+ Chứng minh bất đẳng thức với biến số trung gian này nhƣ thế nào ?

Áp dụng bất đẳng thức Côsi cho 4 số không âm 222 t , 222 t , 222 t ,

262 t , ta có 32 )22( t )62( 2t 1624

62)22.(3 4

422

tt.

Đẳng thức xảy ra khi và chỉ khi 0t .

Ví dụ 5 : Chứng minh rằng:

0,,,

3

133

3

33

3

33

3

zyxxzz

z

zyy

y

yxx

x (1)

Hướng dẫn

Đặt y

zp

z

xn

x

ym ,, 1mnp .

+ Biến đổi (1) thành biểu thức của pnm ,, ?

3

1

11

1

11

1

11

1333

pnm

+ Hãy đặt .;;222 c

abp

b

can

a

bcm Biến đổi (1) thành biểu thức của cba ,, ?

3

11

326

6

326

6

326

6

abcc

c

cabb

b

bcaa

a (2)

Page 58: Sinh viên Nguyễn Thị Lan Hƣơng · PDF fileLỜI CẢM ƠN Lời đầu tiên em xin bày tỏ lòng kính trọng và biết ơn sâu sắc đến Thầy giáo Thạc sĩ Nguyễn

KHÓA LUẬN TỐT NGHIỆP ĐẠI HỌC Nguyễn Thị Lan Hương – K36B SP Toán

54

+ Chúng ta có nhận xét gì về bậc của các phân số trong tổng ở vế trái của (2), và

về vai trò của cba ,, , dấu của mẫu số của các phân số trên?

Các phân số ở trong tổng trên là đồng bậc 0, cba ,, hoán vị vòng quanh,

trong các phân số thì mẫu số đều dƣơng.

+ Nhƣ vậy ta có thể sử dụng bất đẳng thức dạng cộng mẫu số cho (2). Hãy vận

dụng và giải quết bài toán?

Sử dụng dạng cộng mẫu số của bất đẳng thức BunhiaCopski ta có:

cyc

cyc bcaa

cba

bcaa

aVT

326

2333

326

6

2

+ Từ đánh giá này ta suy ra điều gì?

Ta suy ra để chứng minh (1) đúng ta cần chứng minh

cyc

bcaacba3263333

2.2.5. Khai thác các bài toán chứng minh bất đẳng thức

2.2.5.1. Một số bài toán chứng minh bất đẳng thức có nhiều cách giải

Các bài trong nội dung này có sự tổng hợp nhìn bài toán theo nhều

phƣơng diện khác nhau.

Ví dụ 1: Chứng minh

2 2 ; , 0;x a y y a x a x y a với a là hằng số dƣơng.

Hướng dẫn

Cách 1: Lƣợng giác

+ Mối liên hệ giữa các đại lƣợng x và 2a x , giữa y và 2a y đƣợc xác

định nhƣ thế nào?

2 2

2 2 2 2;x a x a y a y a .

+ Với liên hệ này thì các đại lƣợng đƣợc biểu diễn nhƣ thế nào qua các hàm số

lƣợng giác ?

sin.,cos. 2 axaax

cos.,sin. 2 ayaay .

+ Bất đẳng thức cần chứng minh đƣợc chuyển về bất đẳng thức lƣợng giác nào? aaaaa sin.sin.cos.cos.

+ Chứng minh bất đẳng thức lƣợng giác này đƣợc chứng minh nhƣ thế nào?

cos .cos sin .sin 1 os( ) 1c , bất đẳng thức đúng.

Cách 2: Bất đẳng thức Bunhiacopxki

+ Hãy viết lại bất đẳng thức Bunhiacopxki cho hai bộ 2 số

Page 59: Sinh viên Nguyễn Thị Lan Hƣơng · PDF fileLỜI CẢM ƠN Lời đầu tiên em xin bày tỏ lòng kính trọng và biết ơn sâu sắc đến Thầy giáo Thạc sĩ Nguyễn

KHÓA LUẬN TỐT NGHIỆP ĐẠI HỌC Nguyễn Thị Lan Hương – K36B SP Toán

55

22222yxbabyax .

+ Mối liên hệ giữa các đại lƣợng x và 2a x , giữa y và 2a y đƣợc xác

định nhƣ thế nào?

2 2

2 2 2 2;x a x a y a y a .

Các cặp số 2,x a x , 2 ,a y y đều có tổng bình phƣơng không phụ

thuộc vào các biến số.

+ Để trong quá trình đánh giá có đẳng thức này cần sử dụng bất đẳng thức

Bunhiacopxki cho các bộ số nào ?

Áp dụng bất đẳng thức Bunhiacopxki cho hai bộ số 2,x a x và

2 ,a y y , ta có

222222

22 . ayyaxaxyxayax .

axayyax 22 .

Bình luận : Ta vẫn dùng các cặp số ( x , 2a x ), ( y , 2a y ) đều có tổng

bình phương bằng vế phải nhưng khi thay đổi để được kết quả mới.

222222

22 . ayayxaxyaxaxy .

ayaxaxy 22 .

Cách 3: Dùng bất đẳng thức Côsi

+ Hãy viết lại Bất đẳng thức Côsi cho 2 số dƣơng

0,2

BABA

AB .

+ Các số dƣơng x , y có thể coi là căn bậc hai của những số dƣơng nào?

22 , yyxx .

+ Áp dụng bất đẳng thức Côsi cho hai số dƣơng thì vế trái đƣợc đánh giá nhƣ thế

nào ?

axayyax

xayyax

22

222222 .

Cách 4: Bất đẳng thức giữa đƣờng vuông góc và đƣờng xiên trong hình học

O A

B

H

K

Page 60: Sinh viên Nguyễn Thị Lan Hƣơng · PDF fileLỜI CẢM ƠN Lời đầu tiên em xin bày tỏ lòng kính trọng và biết ơn sâu sắc đến Thầy giáo Thạc sĩ Nguyễn

KHÓA LUẬN TỐT NGHIỆP ĐẠI HỌC Nguyễn Thị Lan Hương – K36B SP Toán

56

+ Mối liên hệ giữa x và 2a x đƣợc xác định nhƣ thế nào?

axax 2

22 .

+ Có thể dựng đƣợc không một tam giác vuông có hai cạnh góc vuông bằng x

và 2a x ?

OA = x , OB = aOBxa 222 OAAB .

+ Mối liên hệ giữa y và 2a y đƣợc xác định nhƣ thế nào?

sin AOt

= 21ya

a , cos BOt

= ya

.1

.

+ Có thể dùng hệ thức lƣợng trong tam giác vuông để biểu diễn vế trái qua các

đại lƣợng hình học đƣợc không ?

)(sin..sin..22 AHBKaBOtOBaAOtOAaxayyax

.

+ Dùng bất đẳng thức hình học có thể so sánh biểu thức trên với a đƣợc không ?

aaaABaAHBKa ..)( .

Cách 5: Bất đẳng thức trong đƣờng tròn.

+ Mối liên hệ giữa x và 2a x đƣợc xác định nhƣ thế nào?

axax 2

22 .

+ Có thể dựng đƣợc không một tam giác vuông có hai cạnh góc vuông bằng x và

2a x ?

MA = x , MB = aMBxa 222 MAAB .

+ Mối liên hệ giữa y và 2a y đƣợc xác định nhƣ thế nào?

ayay 2

22 =AB2.

+ Có thể dựng đƣợc hay không một tam giác vuông có hai cạnh góc vuông bằng

y và 2ya , cạnh huyền là AB?

A B

M

N

x

y 2a y

2a x

O

Page 61: Sinh viên Nguyễn Thị Lan Hƣơng · PDF fileLỜI CẢM ƠN Lời đầu tiên em xin bày tỏ lòng kính trọng và biết ơn sâu sắc đến Thầy giáo Thạc sĩ Nguyễn

KHÓA LUẬN TỐT NGHIỆP ĐẠI HỌC Nguyễn Thị Lan Hương – K36B SP Toán

57

+ Vế trái có thể biểu diễn qua các đại lƣợng hình học nhƣ thế nào?

ABMNMBNANBMAxayyax ...22 (Đẳng thức Ptôlêmê).

+ Có thể dùng bất đẳng thức hình học để so sánh biểu thức trên với a đƣợc

không?

aABABMNABMN 2. .

Cách 6: Dùng véc tơ và tọa độ

Trong mặt phẳng toạ độ, cho hai véc tơ ),();,( 2211 yxvyxu

.

+ Biểu thức toạ độ tích vô hƣớng của hai véc tơ trên đƣợc xác định nhƣ thế nào ?

2121.. yyxxvu

.

+ Để vế trái của bất đẳng thức là biểu thức toạ độ của một tích vô hƣớng, cần

chọn véc tơ nhƣ thế nào?

Trƣờng hợp 1 : Chọn );();,( 22 yayavyxu

.

Trƣờng hợp 2 : Chọn );();,( 22 yyavxaxu

.

+ Độ dài các véc tơ đƣợc xác định nhƣ thế nào ?

Trƣờng hợp 1 : 2222 2; yxavyxu

.

Trƣờng hợp 2 : Chọn avu

.

+ Chứng minh bất đẳng thức đã cho bằng tích vô hƣớng nhƣ thế nào ?

vuvu .. .

Chọn trƣờng hợp 2 ta có đpcm.

Bình luận : Ngoài 6 cách trên, ta có thể dùng phƣơng pháp tam thức bậc

hai hoặc biến đổi tƣơng đƣơng để giải bài toán này.

Ví dụ 2: Cho 0,,, 222222 babtzayx .

Chứng minh a) abtzyxtzyxab 2))(())((2 .

b) abtzyxtzyxab 2))(())((2 .

Hướng dẫn

Cách 1: Lƣợng giác

+ Mối liên hệ giữa x và y , giữa z và t đƣợc xác định nhƣ thế nào?

222222 , btzayx .

+ Với liên hệ này thì các số đó đƣợc biểu diễn qua các hàm lƣợng giác nhƣ thế

nào?

sin.,cos.,sin.,cos. btzubyax .

Page 62: Sinh viên Nguyễn Thị Lan Hƣơng · PDF fileLỜI CẢM ƠN Lời đầu tiên em xin bày tỏ lòng kính trọng và biết ơn sâu sắc đến Thầy giáo Thạc sĩ Nguyễn

KHÓA LUẬN TỐT NGHIỆP ĐẠI HỌC Nguyễn Thị Lan Hương – K36B SP Toán

58

+ Bất đẳng thức cần chứng minh đƣợc chuyển về bất đẳng thức lƣợng giác nào ?

ababab 2sincossincossincossincos2

.

ababab 2sincossincossincossincos2 .

+ Bất đẳng thức lƣợng giác đƣợc chứng minh nhƣ thế nào ?

a) ababab 2sin.sincos.cos22

ababab 2cos.22 .

1cos1 (đpcm).

b) ababab 2sin.coscos.sin22

ababab 2sin.22

1sin1 (đpcm).

Cách 2: Véc tơ

Ta chứng minh abtzyxtzyxab 2))(())((2 , bất đẳng thức

còn lại chứng minh tƣơng tự.

Trong mặt phẳng toạ độ, cho hai véc tơ ),();,( 2211 yxvyxu

.

+ Hãy viết lại biểu thức toạ độ tích vô hƣớng của hai véc tơ trên

2121.. yyxxvu

.

+ Để vế trái của bất đẳng thức là biểu thức toạ độ của một tích vô hƣớng cần

chọn véc tơ nhƣ thế nào ?

Trƣờng hợp 1 : Chọn ),();,( tztzvyxyxu

.

Trƣờng hợp 2 : Chọn ),();,( yxtzvtzyxu

.

+ Độ dài các véc tơ đƣợc xác định nhƣ thế nào ?

Trƣờng hợp 1 : btzvayxu 2)(2;22 2222

.

Trƣờng hợp 2 : Chọn 2222, yxtzvtzyxu

.

+ Chứng minh bất đẳng thức đã cho bằng tích vô hƣớng nhƣ thế nào ?

vuvuvu ... .

Chọn trƣờng hợp 1 ta có đpcm.

Cách 3: Bất đẳng thức Bunhiacopxki

+ Hãy viết lại bất đẳng thức Bunhiacopxki cho hai bộ 2 số

22222yxbabyax .

Page 63: Sinh viên Nguyễn Thị Lan Hƣơng · PDF fileLỜI CẢM ƠN Lời đầu tiên em xin bày tỏ lòng kính trọng và biết ơn sâu sắc đến Thầy giáo Thạc sĩ Nguyễn

KHÓA LUẬN TỐT NGHIỆP ĐẠI HỌC Nguyễn Thị Lan Hương – K36B SP Toán

59

+ Mối liên hệ giữa bình phƣơng yx và yx , giữa bình phƣơng của tz và

tz đƣợc xác định nhƣ thế nào?

2222222 ayxyxyx

2222222 btztztz

+ Nhƣ vậy các cặp số yxyx , , tztz , đều có tổng bình phƣơng không

phụ thuộc vào các biến số.

+ Để trong quá trình đánh giá có đẳng thức này, cần sử dụng bất đẳng thức

Bunhiacopxki với các cặp số nào ?

Áp dụng bất đẳng thức Bunhiacopxki cho hai bộ số yxyx , ,

tztz ,

Ta có

22222222242.2)()(.)())(())(( babatztzyxyxtzyxtzyx

abtzyxtzyxab 2))(())((2 .

Ví dụ 3: Cho cbacba ,;0,, . Chứng minh

( ) ( )c a c c b c ab .

Hướng dẫn

Cách 1: Dùng Bất đẳng thức Bunhiacopxki

+ Hãy viết lại bất đẳng thức Bunhiacopxki cho hai bộ 2 số

22222yxbabyax .

+ Mối liên hệ giữa bình phƣơng của c và ca , giữa bình phƣơng của c và

cb đƣợc xác định nhƣ thế nào?

,22

acac ,22

bcbc

+ Nhƣ vậy các cặp số ( c , ca ) và ( c , cb ) đều có tổng bình phƣơng

không phụ thuộc vào các biến số c.

+ Để trong quá trình đánh giá có đẳng thức này thì ta cần sử dụng bất đẳng thức

Bunhiacopxki với các cặp số nào?

Áp dụng Bất đẳng thức Bunhiacopxki cho hai bộ số ( cb , c ) và

( c , ca ), ta có

2

.. ccbcac

22

cac abcbc

22

.

( ) ( )c a c c b c ab .

Cách 2: Lƣợng giác.

+ Hãy xác định mối liên hệ giữa bình phƣơng của c và cb , giữa bình

phƣơng của c và ca ?

Page 64: Sinh viên Nguyễn Thị Lan Hƣơng · PDF fileLỜI CẢM ƠN Lời đầu tiên em xin bày tỏ lòng kính trọng và biết ơn sâu sắc đến Thầy giáo Thạc sĩ Nguyễn

KHÓA LUẬN TỐT NGHIỆP ĐẠI HỌC Nguyễn Thị Lan Hương – K36B SP Toán

60

,22

acac bcbc 22

+ Với liên hệ này thì các đại lƣợng đƣợc biểu diễn nhƣ thế nào qua các hàm

lƣợng giác ?

sin.,cos.,sin.,cos. bcbbcacaac .

+ Bất đẳng thức cần chứng minh đƣợc chuyển về bất đẳng thức lƣợng giác nào ?

abbaab sin..cossin.cos .

+ Chứng minh bất đẳng thức lƣợng giác nhƣ thế nào?

ababab sin..cossin.cos .

1sin..cossin.cos .

sin đpcm.

Cách 3: Sử dụng bất đẳng thức diện tích trong trong hình học.

VT = 1

2AC. BD = SABCD = 2SBCD .BC CD ab .

2.2.5.2. Một số ví dụ về sáng tạo bất đẳng thức

Sáng tạo bất đẳng thức bằng cách nhìn bất đẳng thức đã có theo những

phƣơng diện mới.

Khi sáng tạo bài toán mới, ngƣời ta thƣờng đặc biệt hoá, tổng quát hoá

hoặc mở rộng các kết quả đã có. Tuy nhiên không phải bài toán nào cũng làm

đƣợc nhƣ vây. Phần lớn các bài toán về bất đẳng thức đều có điều kiện của biến

và ta sử dụng yếu tố này để thay đổi phƣơng diện bất đẳng thức, sáng tạo nhiều

bài toán mới. Sau đây là các bài toán minh hoạ.

Ví dụ 1: Cho dcba ,,, là các số thực dƣơng tùy ý.

Chứng minh rằng: 2

222 cba

ac

c

cb

b

ba

a

Nhận xét: Bài toán này đƣợc chứng minh khá đơn giản nhờ đánh giá dựa vào bất

đẳng thức BunhiaCopski.

Sáng tạo bất đẳng thức dựa vào bất đẳng thức ban đầu

B

C A

D

H

c c

a a

b b

Page 65: Sinh viên Nguyễn Thị Lan Hƣơng · PDF fileLỜI CẢM ƠN Lời đầu tiên em xin bày tỏ lòng kính trọng và biết ơn sâu sắc đến Thầy giáo Thạc sĩ Nguyễn

KHÓA LUẬN TỐT NGHIỆP ĐẠI HỌC Nguyễn Thị Lan Hương – K36B SP Toán

61

Ta đặt 6

,3

,2

,t

dz

cy

bxa . Khi đó ta có bất đẳng thức

12

236

36636

2

46

3

2

2 2222 tzyx

xt

t

tz

z

zy

y

yx

x

Bất đẳng thức khá phức tạp vì nó đã đƣợc biến đổi che dấu đi bản

chất bài toán. Có rất nhiều cách đổi biến để sáng tạo bất đẳng thức mới, kỹ

thuật biến đổi càng phức tạp thì vấn đề càng khó đƣợc tìm ra.

Ví dụ 2:Cho các số dƣơng dcba ,,, thoả mãn hệ thức 1abcd .

Hãy chứng minh rằng 21

1

a

21

1

b

21

1

c2

1

12

d.

Đặt t

xd

z

tc

y

zb

x

ya 2222 ;;; , ta phải chứng minh

x

y1

1

y

z1

1

z

t1

12

1

1

t

x.

2

xt

t

tz

z

zy

y

yx

x( bất đẳng thức Nestbit).

Ví dụ 3: Ta có kết quả 3334443 zyxzyxzyx .

Bất đẳng thức này dễ dàng chứng minh vì nó tƣơng đƣơng với

0333333 xzxzzyzyyxyx .

Nhận xét: Bất đẳng thức tƣơng đƣơng với

4 4 4 3 3 3

3x y z x y z

x y z x y z x y z x y z x y z x y z

Bằng cách coi biểu thức là một số azyx

x

; b

zyx

y

; c

zyx

z

,

Ta có bài toán

Chứng minh: 1;,,,3 333444 cbacbacbacba .

Ví dụ 4: Xuất phát. Bất đẳng thức Vasile Cirtoaje

3;0,,2

3

111

zyxzyx

zx

z

yz

y

xy

x.

Thay cba

cz

cba

by

cba

ax

3;

3;

3 0,, cba . Tác giả có một kết quả

rất đẹp và khó.

Đề thi chọn đội dự tuyển toán Đại học Sƣ phạm năm 2007-2008.

Chứng minh bất đẳng thức

Page 66: Sinh viên Nguyễn Thị Lan Hƣơng · PDF fileLỜI CẢM ƠN Lời đầu tiên em xin bày tỏ lòng kính trọng và biết ơn sâu sắc đến Thầy giáo Thạc sĩ Nguyễn

KHÓA LUẬN TỐT NGHIỆP ĐẠI HỌC Nguyễn Thị Lan Hương – K36B SP Toán

62

0,,

)(2

1

)(9)(9)(9 222

cba

cbacbaca

c

cbabc

b

cbaab

a

.

Ví dụ 5: Xuất phát:Với mọi 0,, zyx ta có 2

3222

yx

z

xz

y

zy

x

3 zyx .

Thay c

zb

ya

x1

;1

;1

Ta có bài toán:

Chứng minh rằng2

3

)11

(

1

)11

(

1

)11

(

1

222

ba

cac

bcb

a

3111

;0,, cba

cba

Khi 1abc , thay abc

cab

bca

1

;1

;1

Ta có bài toán

Cho 0,, cba và 1abc . Chứng minh: 3 3 3

1 1 1 3

2a b c b c a c a b

.

Ví dụ 6: Xuất phát từ bất đẳng thức 2cossin nn . Khai triển sinn và

cosn rồi đặt cos =x, sin =y ta có một loạt bất đẳng thức với hai biến x, y

thoả mãn 122 yx .

Bài 1: Cho 122 yx . Chứng minh 22 22 yxxy .

Bài 2: Cho 122 yx . Chứng minh 2)(4)(3 33 yxyx .

Bài 3: Cho 122 yx . Chứng minh )(488 2224 yxxyxx 12 .

Ví dụ 7: Xuất phát từ bất đẳng thức 1coscoscos CBA ABC .

Áp dụng bất đẳng thức cho tam giác có ba góc là 2

;2

;2

BAACCB .

Ta có bài toán

Cho tam giác ABC, chứng minh bất đẳng thức: 12

sin2

sin2

sin CBA

.

Chuyển sang phƣơng diện hình học

Ta có bài toán

Cho tam giác ABC ngoại tiếp đƣờng tròn (I,r). Chứng minh bất đẳng thức

rICIBIA

1111 .

Ví dụ 8: Xuất phát: 32

tan2

tan2

tan CBA

ABC .

Page 67: Sinh viên Nguyễn Thị Lan Hƣơng · PDF fileLỜI CẢM ƠN Lời đầu tiên em xin bày tỏ lòng kính trọng và biết ơn sâu sắc đến Thầy giáo Thạc sĩ Nguyễn

KHÓA LUẬN TỐT NGHIỆP ĐẠI HỌC Nguyễn Thị Lan Hương – K36B SP Toán

63

Sử dụng công thức xxx

x

xx

x

xcot

sin

1

sin

cos1

2cos.

2sin2

2sin2

2tan

2

Ta có bài toán

Chứng minh bất đẳng thức: 3cotcotcotsin

1

sin

1

sin

1 CBA

CBA

ABC

Nhận xét: Áp dụng công thức S

cbaCBA

4cotcotcot

222 ;

S

bc

A 2sin

1 ;

S

ca

B 2sin

1 ;

S

ab

C 2sin

1

Ta có bài toán: Chứng minh Scbacabcab 34222 222 ABC .

Page 68: Sinh viên Nguyễn Thị Lan Hƣơng · PDF fileLỜI CẢM ƠN Lời đầu tiên em xin bày tỏ lòng kính trọng và biết ơn sâu sắc đến Thầy giáo Thạc sĩ Nguyễn

KHÓA LUẬN TỐT NGHIỆP ĐẠI HỌC Nguyễn Thị Lan Hương – K36B SP Toán

64

KẾT LUẬN

Trong chƣơng trình Toán phổ thông, bất đẳng thức đóng vai trò hết sức

quan trọng. Tuy nhiên, trong thực tế rất nhiều học sinh học bất đẳng thức còn

chƣa tốt, còn gặp nhiều khó khăn trong việc đi tìm lời giải bài toán, điều đó làm

cho các em không hứng thú với chúng. Mặt khác, nhiều giáo viên phổ thông cũng

còn lúng túng trong phƣơng pháp giảng dạy bất đẳng thức, chƣa vận dụng tốt các

phƣơng pháp dạy học tích cực vào giảng dạy, càng làm cho việc dạy học chƣa

đạt đƣợc hiệu quả cao. Trƣớc vấn đề đặt ra, trong khóa luận của mình, tôi đã nêu

ra:

- Sự cần thiết phải đổi mới phƣơng pháp giảng dạy các bài toán chứng

minh bất đẳng thức nhờ vào các phƣơng pháp dạy học tích cực, trong đó, chủ yếu

là hai phƣơng pháp thƣờng đƣợc vận dụng ở trƣờng phổ thông mang lại hiệu quả

cao là phƣơng pháp gợi mở vấn đáp và phƣơng pháp dạy học phát hiện và giải

quyết vấn đề.

- Hệ thống các câu hỏi gợi mở cho từng dạng bài toán chứng minh bất

đẳng thức, qua đó giúp các em trƣớc hết là nhận dạng bài toán, phát hiện và giải

quyết vấn đề đặt ra.

- Hệ thống bài tập cho các em thực hành vận dụng, nhằm tạo lập niềm tin

vào khả năng giải quyết bài toán trƣớc mắt, bên cạnh đó là những nhận xét nhỏ

sau mỗi dạng toán góp phần khắc sâu kiến thức, giúp các em dần hình thành kỹ

năng biết tự rút ra đƣợc những kinh nghiệm cho bản thân, giúp các em thấy đƣợc

vẻ đẹp của bất đẳng thức, ham thích, say mê nghiên cứu, tìm tòi, sáng tạo, giải

đƣợc các bài toán chứng minh bất đẳng thức, khám phá, khai thác sâu hơn các

bài toán đó.

Do thời gian cũng nhƣ kiến thức và kinh nghiệm của bản thân còn hạn

chế, mặc dù đã có nhiều cố gắng nhƣng không thể tránh khỏi những thiếu sót.

Tôi rất mong nhận đƣợc sự nhận xét, góp ý của các Thầy cô, cũng nhƣ của các

bạn để khóa luận của tôi đƣợc hoàn thiện hơn.

Em xin chân thành cảm ơn!

Hà nội, ngày 10 tháng 5 năm 2014

Page 69: Sinh viên Nguyễn Thị Lan Hƣơng · PDF fileLỜI CẢM ƠN Lời đầu tiên em xin bày tỏ lòng kính trọng và biết ơn sâu sắc đến Thầy giáo Thạc sĩ Nguyễn

KHÓA LUẬN TỐT NGHIỆP ĐẠI HỌC Nguyễn Thị Lan Hương – K36B SP Toán

65

TÀI LIỆU THAM KHẢO

1. Nguyễn Văn Hà (1999), Phương pháp toán sơ cấp, NXB ĐHSP Hà Nội 2.

2. Phạm Kim Hùng (2006), Sáng tạo Bất đẳng thức, NXB Tri thức.

3. Nguyễn Bá Kim (2004), Phương pháp dạy học môn Toán , NXB ĐHSP.

4. Trần Thị Tuyết Oanh (Chủ biên) – Phạm Khắc Chƣơng – Phạm Viết Vƣợng –

Bùi Minh Hiền – Nguyễn Ngọc Bảo – Bùi Văn Quân – Phan Hồng Vinh – Từ

Đức Văn (2012), Giáo trình Giáo dục học – Tập 1, NXB ĐHSP.

5. Trần Thị Tuyết Oanh – Phạm Khắc Chƣơng – Phạm Viết Vƣợng – Nguyễn

Văn Diện – Lê Tràng Định (2013), Giáo trình Giáo dục học – Tập 2, NXB

ĐHSP.

6. Ngô Thế Phiệt (2007), Một số phương pháp mới trong chứng minh bất đẳng

thức, NXB GD.

7. Trần Phƣơng (2000), Các phương pháp và kỹ thuật chứng minh bất đẳng

thức, NXB TP HCM.

8. Trần Phƣơng (2012), Những viên kim cương trong bất đẳng thức toán học,

NXB

Tri Thức.